p3 Modul TOEFL

Download as pdf or txt
Download as pdf or txt
You are on page 1of 93

PREFACE

This module is designed for students studying Test of English as Foreign Language (TOEFL)
Preparation at Faculty of Social and Political Sciences Pasundan University. The material in this
module was specifically chosen to assist students in advancing their Test of English as Foreign
Language (TOEFL) score. It includes lots of interesting stories and has exercise that include
Structure, Reading Comprehension, and Listening Comprehension.
Finally, I would like to express gratitude for all people who has participated in compiling this
module by various materials included.

Bandung, February 2021

The Compiler

i
CONTENTS

Preface i

Chapter 1 Structure: Subject - Verb & Object of Preposition 1


Reading Comprehension : Main Ideas 5

Chapter 2 Present Participle & Past Participle 8


Reading Comprehension: Stated Detail Question 13

Chapter 3 Structure: Coordinate Connectors & Adverb Clause Connectors 16


Reading Comprehension : References and Vocabularies 20

Chapter 4 Structure: Noun Clause Connectors 22


Reading Comprehension: Reading Practices 25

Chapter 5 Structure: Adjective Clause Connectors 29


Reading Comprehension: Reading Practices 33

Chapter 6 Structure: Agreements 35


Reading Comprehension: Reading Practices 41

Chapter 7 Structure: Parallel Structures 43


Reading Comprehension: Inference Questions 48

Chapter 8 Structure: Participles after Have and Be, Base Form Verbs after 51
Modals
Reading Comprehension: ‘Where’ Questions 55

Chapter 9 Listening Comprehension: Short Dialogues (Skill 1–8) 57

Chapter 10 Listening Comprehension: Long Talks (Skill 9–15) 74

ii
CHAPTER 1

Structure: Subject - Verb & Object of Preposition


Reading: Main Ideas
A. STRUCTURE
The structure questions:

Multiple choice question is a type of question that tests your knowledge of the correct
structure of English sentences appear on the paper TOEFL test.

Example:

Homo sapiens about 40,000 years ago.


(A) their first appearance
(B) first appeared
(C) first appearing
(D) they first appeared

In this example, you should notice that the sentence has a subject ‘Homo sapiens’ but needs a
verb. Answer (B) is the correct answer because it contains the verb ‘appeared’. Answers (A) and
(C) are incorrect because they do not have verbs, and answer (D) is incorrect because it has an
extra subject. You should therefore choose answer (B).

From the example above, we can conclude that the ability of analyzing the completion of the
sentence is very important. You need to be able to differentiate a sentence based on its words’
formation.

Tips for the structure questions:

1. First, study the sentence. Your purpose is to determine what is needed to complete the
sentence correctly.
2. Then study each answer based on how well it completes the sentence. Eliminate answers that
do not complete the sentence correctly.
3. Do not try to eliminate incorrect answers by looking only at the answers. The incorrect answers
are generally correct by themselves. The incorrect answers are generally incorrect only when
used to complete the sentence.

I. SUBJECTS AND VERBS

A sentence in English should have a subject and a verb. The most common types of problems that
you will encounter in structure questions on the TOEFL test are related to subjects and verbs perhaps
the sentence is missing either the subject or the verb or both, or perhaps the sentence has an extra
subject or verb.

1
Example 1:

was ringing continuously for hours.


A. Loudly B. In the morning C. The phone D. The bells

In this example, you should notice immediately that there is a verb was ringing, but there is no
subject. Answer © is the best answer because it is a singular subject that agrees with the singular
verb ‘was ringing”. Answer (A) ‘loudly’ and answer (B) ‘in the morning’ are not subjects, so
they are not correct. Although answer (D) ‘bells’ could be a subject, it is not correct because
‘bells’ is plural and it does not agree with the singular verb was ringing.

Example 2:

Newspapers every morning and every evening.


A. delivery B. are delivered C. on time D. regularly

In this example, you should notice immediately that the sentence has a subject ‘newspapers’ but
that there is no verb. Because answer (B) ‘are delivered’ is a verb, it is the best answer. Answers
(A), (C), and (D) are not verbs, so they are not correct.

Example 3:

The plane landing at the airport in five minutes.


A. it is B. it really is C. is descending D. will be

This sentence has a subject ‘plane’ and has part of a verb ‘landing’. To be correct, some form of the verb
be is needed to make the verb complete. Answers (A) and (B) are incorrect because the sentence already
has a subject ‘plane’ and does not need the extra subject ‘it’. Answer (C) is incorrect because
‘descending’ is an extra part of a verb that is unnecessary because of ‘landing’. Answer (D) is the best
answer; ‘will be’ together with ‘landing’ is a complete verb.

The key information that you should remember about subjects and verbs:

A sentence in English must have at least one subject and one verb. The first thing you should
do as you read a sentence in the structure section of the TOEFL test is to find the
subject and the verb.

2
PRACTICE 1

Underline the subjects once and the verbs twice in each of the following sentences. Then indicate if
the sentences are correct (C) or incorrect (I).

1. My best friend always helpful with problems. ( I )


2. The bus schedule has changed since last week. ( C )
3. Accidentally dropped the glass on the floor. ( )
4. The customer paying the clerk for the clothes. ( )
5. The professor handed the syllabus to the students. ( )
6. Each day practiced the piano for hours. ( )
7. The basketball player tossed the ball into the hoop. ( )
8. The new student in the class very talkative and friendly. ( )
9. Walking with the children to school. ( )
10.The whales headed south for the winter. ( )

II. OBJECTS OF PREPOSITIONS

An object of a preposition is a noun or a pronoun that comes after a preposition such as in, at, of, to,
by, behind, and on to form a prepositional phrase.

Example 1:

The trip (to the island) (on Saturday) will last (for three hours).

This sentence contains three objects of prepositions. ‘Island’ is the object of the preposition ‘to’,
‘Saturday’ is the object of the preposition ‘on’, ‘hours’ is the object of the preposition ‘for’. An object
of a preposition can cause confusion in the structure section of the TOEFL test because it can be
mistaken for the subject of a sentence.

Example 2:

To Mike _______ was a big surprise.


A. really B. the party C. funny D. when

In this example, you should look first for the subject and the verb. You should notice the verb ‘was’
and should also notice that there is no subject. Do not think that Mike is the subject, ‘Mike’ is the
object of the preposition ‘to’, and cannot be both a subject and an object at the same time. Because a
subject is needed in this sentence, answer (B) the party is the best answer. Answers (A), (C), and (D)
are not correct because they cannot be subjects.

The key information that you should remember about objects of prepositions:

A preposition is followed by a noun or pronoun that is called an object of the preposition.


If a word is an object of a preposition, it is not the subject.

3
PRACTICE 2

Each of the following sentences contains one or more prepositional phrases. Underline the subjects
once and the verbs twice. Circle the prepositional phrases that come before the verb. Then indicate if
the sentences are correct (C) or incorrect (I).

1. The name of the baby in the crib is Jack. ( C )


2. By the next meeting of the class need to turn in the papers. ( )
3. The directions to the exercise on page twenty unclear. ( )
4. Because of the heavy coin throughout the night, the walkways are muddy. ( )
5. During the week eat lunch in the school cafeteria. ( )
6. In the morning after the concert was tired. ( )
7. In the summer the trip to the mountains is our favorite trip. ( )
8. In a box on the top shelf of the cabinet in the hallway of the house. ( )
9. With her purse in her hand ran through the door. ( )
10. At1:00 in the morning the alarm clock on the table beside the bed rang. ( )

PRACTICE 3

Underline the subjects once and the verbs twice. Circle the prepositional phrases that come before
the verb. Then indicate if the sentences are correct (C) or incorrect (I)

1. During the meeting in the office discussed the schedule. ( )


2. The doctor gave the patient a prescription. ( )
3. The tall evergreen trees along the road. ( )
4. The watch in the jewelry box needs a new battery. (_ _)
5. Pleasantly greets everyone in all the offices every morning. (_ )
6. In the office of the building across the street from the park on the corner. ( )
7. The dishes in the sink really need to be washed as soon as possible. ( )
8. In a moment of worry about the problem with the cash in the account. ( )
9. The plane from New York circling the airport. ( )
10. On a regular basis the plants in the boxes under the window in the kitchen are
watered and fed. ( )

PRACTICE 4

Choose the letter of the word or group of words that best completes the sentence.

1. Mark Twain the years after the Civil War the ‘Gilded Age.’
A. called B. calling C. he called D. his calls

2. Early toes instead of hooves on their feet.


A. horses B. had horses C. horses had D. horses having

3. grow close to the ground in the short Arctic summer.


A. Above tundra plants C. Tundra plants are found
B. Tundra plants D. For tundra plants

4
4. In 1867, Alaska from the Russians for $7.2 million.
A. purchased the United States C. the United States’ purchase of
B. to purchase the United States D. the United States purchased

5. Between 1725 and 1750, New England witnessed an increase in the specialization of
.
A. occupations B. occupies C. they occupied D. it occupied them

6. The large carotid artery to main parts of the brain.


A. carrying blood C. carriers blood B. blood is carried D. blood carries

7. Radio as the first practical system of wireless telegraphy.


A. Marconi’s development C. Developing Marconi
B. The development by Marconi D. Marconi developed

8. In 1975, the first successful space probe to beginning to send information back to
Earth.
A. Venus B. Venus the C. Venus was D. Venus it was

9. The two biggest resort Arkansas are Hot Springs and Eureka Springs.
A. in B. towns in C. towns are D. towns are in

10. NASA’s Lyndon B. Johnson Space Center control center for the Mercury,
Gemini, and Apollo space flights.
A. it was at the B. it was the C. was the D. the

A. READING

Reading for Main Ideas

Almost every reading passage on the paper TOEFL test, will have a multiple choice question about
the main idea of a passage. Such a question may be worded in a variety of ways; you may, for ex-
ample, be asked to identify the topic, subject, title, primary idea, or main idea. These questions are all
really asking what primary point the author is trying to get across in the passage. Since TOEFL,
passages are generally written in a traditionally organized manner, it is relatively easy to find the main
ideas by studying the topic sentences, which are most probably found at the beginning of each
paragraph.
If a passage consists of only one paragraph, you should study the beginning of that paragraph to
determine the main idea. Look at a multiple choice e x a m p l e from the paper TOEFL test that asks
about the topic of a passage with one paragraph.

Example:

The passage

Basketball was invented in 1891 by a physical education instructor in Springfield, Massachu-


setts, by the name of James Naismith. Because of the terrible weather in winter, his physical
education students were indoors rather than outdoors. They really did not like the idea of boring,
repetitive exercises and preferred the excitement and challenge of a game. Naismith figured out
a team sport that could be played indoors on a gymnasium floor, that involved a lot of running,
that kept all team members involved, and that did not allow the tackling and physical contact
of American-style football.

5
The question

What is the topic of this passage?

A. The life of James Naismith


B. The history of sports
C. Physical education and exercise
D. The origin of basketball

The first sentence of this passage states that basketball was invented, so the invention of
basketball is probably the topic. A quick check of the rest of the sentences in the passage con-
firms that the topic is, in fact, the beginnings of the sport of basketball.
Now you should check each of the answers to determine which one comes closest to the topic
that you have determined. Answer (A) mentions James Naismith but not basketball, so it is not
the topic. Answer (B) is too general; it mentions sports but does not mention basketball. Answer
(C) is also too general; it mentions physical education but does not mention basketball. The best
answer is therefore answer (D): the origin of basketball means that the way that basketball was
invented is going to be discussed.
If a passage consists of more than one paragraph, you should study the beginning of each
paragraph to determine the main idea.

HOW TO IDENTIFY THE QUESTION?

 What is the topic of the passage?


 What is the subject of the passage?
 What is the main idea of the passage?
 What is the author’s main point in the passage?
 With what is the author primarily concerned?
 Which of the following would be the best title?

WHERE TO FIND THE ANSWER

The answer to this type of question can generally be determined by looking at the first sentence of
each paragraph.

HOW TO ANSWER THE QUESTION?

1. Read the first line of each paragraph.


2. Look f or a common theme or idea in the first lines.
3. Pass your eyes quickly over the rest of the passage to check that you have really found the
topic sentence(s).
4. Eliminate any definitely wrong answers and choose the best answer from the remaining
choices.

6
PRACTICE 5
Most of the ice on the Earth, close to 90 percent of it is covering the surface of the continent
Antarctica. It does not snow very much in Antarctica, but whatever snow does fall remains and
grows deeper and deeper. In some areas of Antarctica, the ice has perhaps been around for as much
as a million years and now is more than two miles deep.

1. The main idea of the passage is that ….


A. the Earth is a cold planet
B. most of the Earth’s ice is found in Antarctica
C. it snows more in Antarctica than in any other place on the Earth
D. Antarctica is only two miles wide but is 90 percent ice

2. The best title for passage is ….


A. Snowfall in Antarctica
B. The Icy Earth
C. The Cold, Cold Snow
D. The Causes of Antarctica’s Ice Pack

The extremely hot and humid weather that occurs in the United States in July and August is commonly
called the “dog days” of summer. This name comes from the star Sirius, which is known as the Dog
Star. Sirius is the brightest visible star, and in the hot summer months it rises in the east at approximately
the same time as the Sun. As ancient people saw this star rising with the Sun when the weather was at
its hottest, they believed that Sirius was the cause of the additional heat; they believed that this bright
star added its heat to the heat of the Sun and that these two together made summer weather so
unbearably hot.

3. The topic of this passage is ….


A. how dogs like to play during the summer
B. the causes of hot and humid weather
C. why the hot summer days are known as the “dog days”
D. the days that dogs prefer

4. The main idea of this passage is that ….


A. the name for the summer days came from Sirius, the Dog Star
B. dogs generally prefer hot summer days
C. the hottest days occur in the summer because of the movements of the Sun
D. Sirius rises at almost the same time as the Sun during the summer months

7
CHAPTER 2
Present Participle & Past Participle
Reading: Stated Detail Question
A. STRUCTURE

I. PRESENT PARTIPLE

Present participles can cause confusion in the structure questions on the TOEFL test because a
present participle can be either an adjective or a part of the verb. A present participle is the -ing
form of the verb. It is part of the verb when it is preceded by some form of the verb be.

Example 1:

The train is arriving at the station now.

VERB

In this sentence ‘arriving’ is part of the verb because it is accompanied by is. A present
participle is an adjective when it is not accompanied by some form of the verb be.

Example 2:

The train arriving at the station now is an hour late.


ADJECTIVE

In this sentence, ‘arriving’ is an adjective and not part of the verb because it is not accompanied
by some form of be. The verb in this sentence is ‘is’.

The following example shows how a present participle can be confused with the verb in
structure questions on the TOEFL test.

Example 3:

The film …. appearing at the local theater is my favorite.


A. now B. is C. it D. was

In this example, if you look at only the first words of the sentence, it appears that ‘film’ is the
subject and ‘appearing’ is part of the verb. If you think that ‘appearing’ is part of the verb, you
might choose answer (B) ‘is’ or answer (D) ‘was’ to complete the verb. However, these two
answers are incorrect because ‘appearing’ is not part of the verb. You should recognize that
‘appearing’ is a participial adjective rather than a verb because there is another verb in the
sentence ‘is’.
In this sentence, there is a complete subject ‘film’ and a complete verb ‘is’, so this sentence does
not need another subject or verb. The best answer to this question is answer (A).

8
The key information that you should remember about present participles:

A present participle is the-ing form of the verb. The present participle can be part of the Verb
or an adjective. It is part of the verb when it is accompanied by some form of the verb be. It
is an adjective when it is not accompanied by some form of the verb be.

PRACTICE 1:

Each of the following sentences contains one or more present participles. Underline the subjects
once and the verbs twice. Circle the present participles and label them as adjectives or verbs.
Then indicate if the sentences are correct (C) or incorrect (I).

1. The crying baby needs to be picked up. (C) ADJ


2. The clothes are lying on the floor should go into the washing machine. (I) VERB
3. The waitress bringing the steaming soup to the waiting diners. ( )
4. Most of the striking workers are walking the picket line. ( )
5. For her birthday, the child is getting a talking doll. ( )
6. The setting sun creating a rainbow of colors in the sky. ( )
7. The ship is sailing to Mexico is leaving tonight. ( )
8. The letters needing immediate answers are on the desk. ( )
9. The boring class just ending a few minutes ago. ( )
10. The fast-moving clouds are bringing freezing rain to the area. ( )

II. PAST PARTICIPLE

Past participles can cause confusion in structure questions on the TOEFL test because a past
participle can be either an adjective or a part of the verb. The past participle is the form of the
verb that appears with have or be. It often ends in-ed, but there are also many irregulars past
participles in English.

EXAMPLES:

1. The mailman has left a letter in the mailbox.


VERB

2. The classes were taught by Professor Smith.


VERB

In the first sentence, the past participle left is part of the verb because it is accompanied by
has.
In the second sentence, the past participle taught is part of the verb because it is accompanied
by were.

9
A past participle is an adjective when it is not accompanied by some form of be or have.

EXAMPLES:

3. The letter left in the mailbox was for me.


ADJECTIVE

4. The classes taught by Professor Smith were very interesting.


ADJECTIVE

In the third sentence, ’left’ is an adjective rather than a verb because it is not accompanied by a
form of ‘be’ or ‘have’ (and there is verb ‘was’ later in the sentence).
In the second sentence, ‘taught’ is an adjective rather than a verb because it is not accompanied
by a form of be or have (and there is a verb ‘were’ later in the sentence).
The following example shows how a past participle can be confused with the verb in structure
questions on the TOEFL test.

EXAMPLE 5:

The bread baked this morning smelled delicious.


A. has B. was C. it D. just

In this example, if you look only at the first few words of the sentence, it appears that bread is
the subject and baked is either a complete verb or a past participle that needs a helping verb. But
if you look further in the sentence; you will see the verb smelled. You will then recognize that
baked is a participial adjective and is therefore not part of the verb. Answers (A) and (B) are
incorrect because baked is an adjective and does not need a helping verb such as ‘has’ or ‘was’.
Answer (C) is incorrect because there is no need for the subject it. Answer (D) is the best answer
to this question.

The key information that you should remember about past participles:

A past participle often ends in-ed, but there are also many irregulars past participles. For many
verbs, including-ed verbs, the simple past and the past participle are the same and can be easily
confused. The -ed form of the verb can be the simple past, the past participle of a verb, or an
adjective.

PRACTICE 2:

Each of the following sentences contains one or more past participles. Underline the subjects
once and the verbs twice. Circle the past participles and label them as adjectives or verbs. Then
indicate if the sentences are correct (C) or incorrect (I).

1. .The food is served in this restaurant is delicious. ( I )

VERB
2. The plane landed on the deserted runway. ( C )

ADJ

10
3. The unexpected guests arrived just at dinner time. ( )
4. The courses are listed in the catalogue are required courses. (______)
5. The teacher found the lost exam. ( )
6. The small apartment very crowded and disorganized. ( )
7. The photographs developed yesterday showed Sam and his friends. ( )
8. The locked drawer contained the unworn jewels. ( )
9. The tree was blown over in the storm was cut into logs. ( )
10. The students registered in this course are listed on that sheet of paper. ( )

PRACTICE 3:

1. Our hosts are serving drinks on the tiled patio. ( )


2. The tired woman taking a much-needed nap. ( )
3. The letters were sent on Monday arrived on Wednesday. ( )
4. The winners deserved the big prize. ( )
5. The plants are growing in the garden need a lot of water. ( )
6. The shining stars lit up the darkened sky. ( )
7. The driver rapidly increased the speed of the racing car. ( )
8. The excited children trying to build a snowman in the falling snow. ( )
9. The students are completing the course will graduate in June. ( )
10.The dissatisfied customer is returning the broken toaster to the store. ( )

PRACTICE 4:

Choose the letter of the word or group of words that best completes the sentence.

1. The f i r s t appeared during the last period of the dinosaurs’ reign.


A. flowers are plants B. plants have flowers
C. plants flowers D. flowering plants

2. The earliest medicines from plants of various sorts.


A. obtaining B. they obtained
C. were obtained D. they were obtained

3. Simple sails were made from canvas over a frame.


A. a stretch B. stretched C. was stretched D. it was stretched

4. Pluto’s moon Charon in a slightly elliptical path around the planet.


A. moving B. is moving C. it was moving D. in its movement

5. Techniques of breath control form art essential part of any program to improve the
voice.
A. it trains B. train C. trains D. training

6. Robert E. Lee the Confederate Army to General Grant in 1865 at the Appomattox
A. surrendered B. he surrendered C. Surrendering D. surrender

7. The p i t u i t a r y gland, the brain, releases hormones to control other glands.


A. found below B. it is found below
C. its foundation below D. finds itself below

11
8. At around two years of age, many children regularly produce sentences three or
four words.
A. are containing B. containing C. contain D. contains

9. Multinational companies it increasingly important to employ internationally


acceptable brand names.
A. finding B. are finding C. they find D. they are finding

10. The cornea is located under the conjunctiva, on of the eye.


A. the part is exposed B. exposed the part
C. the exposed part D. exposes the part

PRACTICE 5

Choose the letter of the word or group of words that best completes the sentence.

1. first settled the Hawaiian Islands between A.D 300 and 750.
A. The Polynesians B. The Polynesians arrived
C. Because of the Polynesians D. It was the Polynesians

2. In 1066, a bright comet in the sky attracted much attention.


A. was appearing B. appears C. it appeared D. appearing

3. In some daguerreotype cameras, through a hole in the back of the box.


A. the object’s view B. the object was viewed
C. from the view of the object D. viewed the object

4. In the Stone Age, stone tools with other rock materials.


A. polish in B. they polished C. for polish D. were polished

5. The first steamship to cross the Atlantic Savannah, in 1819.


A. was the B. it was the C. the D. in it the

6. The Earth’s plates meet each other at cracks in the Earth faults.
A. were called B. calls C. called D. it was called

7. The first plant-like organisms probably in the sea, perhaps three billion years ago.
A. life B. living C. lived D. it was living

8. In Watch the Skies, Curtis Peebles attempt to explain America’s belief in fly-
ing saucers.
A. makes a fascinating B. making a fascinating
C. fascination with making D. fascination made a

9. The irregular coastline of a succession of bays and inlets, with the hook of the
Cape Cod peninsula in the southeast.
A. Massachusetts B. Massachusetts is
C. Massachusetts it is D. Massachusetts on

12
READING FOR DETAILS

Detail questions ask you about specific information in the passage and what is not in the passage
or not true according to the passage.

QUESTIONS USUALLY APPEAR ON READING FOR DETAILS

 According to the passage …


 It is stated in the passage …
 The passage indicates that …
 Which of the following is true?
 Which of the following is NOT mentioned in the passage?
 According to the passage, all of the following are true EXCEPT …

HOW TO ANSWER THE QUESTION?

1. The answers to detail questions will follow the order of information presented in the passage.
2. The correct answers to detail questions are often a restatement of what is stated in the passage.
3. If the question has the word NOT or EXCEPT, choose the answer that is not true or not mentioned
in the passage.

Example:

The Passage

Flutes have been around for quite some time, in all sorts of shapes and sizes and made from a variety
of materials. The oldest known flutes are about 20,000 years old: they were made from hollowed-
out bones with holes cut in them. In addition to bone, older flutes were often constructed from
bamboo or hollowed-out wood.

Today’s flutes are generally made of metal, and in addition to the holes they have a complicated
system of keys, levers, and pads. The instrument belonging to well-known flautist James Galway
is not just made of any metal, it is made of gold.

The Questions

1. According to the passage, the oldest flutes …


A. had holes cut in them
B. were made of metal
C. were made 200,000 years ago
D. had a complicated set of levers and pads

DISCUSSION:

Since this question asks about the oldest flutes, you should see that this question is answered in
the second sentence. The passage states that the oldest known flutes were bones with holes cut in
them, so the best answer is answer A. Answers B and D are true about today’s flutes but not about
the oldest flutes, so they are incorrect. Answer C is an incorrect number; the oldest flutes are
20,000 years old, not 200,000 years old.

13
2. Which of the following is NOT mentioned in the passage?
A. All flutes made of metal
B. There was a flute made of gold
C. Flutes have uneasy system of keys, lever, and pads.
D. James Galway is a famous flutist.

DISCUSSION:
The question is intended to find out the answer which is not informed by the passage. Answer B, C and
D are obviously stated in the passage. Although answer A seems correct, but the world generally in the
passage is not the restatement of the word all in answer A. Then, Answer A is the correct answer to
choose.

PRACTICE 6

Many parts of the south western United States would become deserts again without the waters of the
Colorado River. A system of thousands of miles of canals, hundreds of miles of tunnels and aque-
ducts, and numerous dams and reservoirs bring Colorado River water to the area. The Imperial Val-
ley in southern California is an example of such a place; it is a vast and productive agricultural area
that was once a desert. Today, 2,000 miles of canals irrigate the fertile land and keep it productive.

1. Which of the following is mentioned in the passage as a way that Colorado River water gets to
the Southwest?
A. By truck B. In bottles C. In wells D. Through canals

2. According to the passage, the Imperial Valley


A. is a desert today B. is located in Colorado
C. produces a lot of agricultural goods D. does not require irrigation

PRACTICE 7
The plane with the largest wingspan ever built was nicknamed the Spruce Goose. The
wingspan of the Spruce Goose was 320 feet (almost 100 meters), and the plane weighed 200
tons. It was so big that it needed eight engines to power it.
The plane was designed by Howard Hughes in response to a U.S. government request for a
(5) plane that was able to carry a large cargo for the war effort. It was made of wood because
wood is a less critical material in wartime than metal.
The plane was so difficult to build that it never really got used. It was flown one time only by
Hughes himself, on November 2, 1947; during that flight it traveled a distance of less than one
mile over the Los Angeles Harbor, but it did fly. Today, the Spruce Goose is an exhibit for the
(10) public to see in Long Beach, California.

1. The passage indicates that the plane was designed


A. as a cargo plane B. as a racing plane C. to carry wood D. for exhibition

2. According to the passage, the Spruce Goose is constructed from


A. wood B. light weight metal C. Plastic D.steel

3. According to the passage, when the Spruce Goose flew,


A. it went on a short distance B. it fell into the Los Angeles Harbor
C. it flew 100 miles D. it carried a large cargo

14
4. The passage indicates that the Spruce Goose today
A. flies regularly for the U.S. government B. is in the Los Angeles Harbor
C. is in storage D. can be seen by the public

PRACTICE 8
The ancestors of humans had a lot of more hair than the human of today. In fact, they had thick hair all
over their bodies. This thick hair was necessary for protection against the cold of the Ice Ages. As Earth
got warmer, the hair began to thin out, except for on the head. The head hair has remained through the
evolutionary process, both as a sort of pillow to cushion the sensitive head when it gets banged around
and as a sort of hat to keep the head warm and prevent so much heat from escaping through the scalp.

1. According to the passage, what happened as the temperature on Earth increased?


A. The hair on the head began to thin out. B. The hair on the body remained the same.
C. The hair on the body got thicker. D. The hair on the body began to thin out.

2. The author indicates that one of the purposes of hair on the head is to
A. fill up pillows B. help heat escape through the scalp
C. ensure that the head is warm D. make it easier to think

15
CHAPTER 3
Coordinate Connectors & Adverb Clause Connectors
Reading: References and Vocabularies

A. STRUCTURE

I. COORDINATE CONNECTORS

Many sentences in English have more than one clause. A clause is a group of words containing a
subject and a verb. When you have two clauses in an English sentence, you must connect the two
clauses correctly. One way to connect two clauses is to use and, but, or, or, so between the clauses.

EXAMPLES:

1. The sun was shining, and the sky was blue.


2. The sky was blue, but it was very cold.
3. It may rain tonight, or it may be clear.
4. It was raining outside, so I took my umbrella.

In each of these examples, there are two clauses that are correctly joined with a coordinate
connector ‘and, but, or, or, so’ and a comma (,).

The following example shows how this sentence pattern could be tested in structure questions
on the TOEFL test.

5. I forgot my coat, I got very cold.


(A) then (B) so (C) later (D) as a result

In this example you should notice quickly that there are two clauses, I forgot my coat and I got
very cold. This sentence needs a connector to join the two clauses. Then, later, and as a result are
not connectors, so answers (A), (C), and (D) are not correct. The best answer is answer (B) because
so can connect two clauses in this manner.

The following chart lists the coordinate connectors and the sentence pattern used with them.

COORDINATE CONNECTORS

and, but, or, so

S V, Coordinate S V
connector

It was raining, but Bill went out to play

16
PRACTICE 1

Each of the following sentences contains more than one clause. Underline the subjects once and the
verbs twice. Circle the connectors. Then indicate if the sentences are correct (C) or incorrect (I).

1. C The lawn needs water every day, or it will turn brown.


2. I The book was not long, but it difficult to read.
3. It was raining, so decided not to go camping.
4. The material has been cut, and the pieces have been sewn together.
5. The patient took all the medicine, he did not feel much better.
6. The bill must be paid immediately, or the electricity will turned off.
7. The furnace broke so the house got quite cold.
8. The dress did not cost too much, but the quality it seemed excellent.
9. The leaves kept falling off the trees, arid the boys kept raking them up, but
the yard was still covered.
10. The postman has already delivered the mail, so the letter is not going to
arrive today, it probably will arrive tomorrow.

II. ADVERB CLAUSE CONNECTORS


Sentences with adverb clauses have two basic patterns in English. Study the clauses and
connectors in the following sentences:

- He is tired because he has been working hard.


- Because he has been working hard, he is tired

In each of these examples, there are two clauses:


1. he is tired
2. he has been working hard.

The clause he has been working hard is an adverb clause that is introduced with the connector
because. In the first example, the connector ‘because’ comes in the middle of the sentence, and no
comma (,) is used. In the second example, the connector ‘because’ comes at the beginning of the
sentence. In this pattern, when the connector comes at the beginning of the sentence, a com- ma
(,) is required in the middle of the sentence.

Example:

arrived at the library, he started to work immediately.

(A) The student


(B) When
(C) He
(D) After the student

17
In this example you should recognize easily that the verb ‘arrived’ needs a subject. There is an
other clause, he started to work immediately. If you choose an answer (A) or answer (C), have a
subject for the verb ‘arrived’, but you will not have a connector to join the two clauses. Because
you need a connector to join two clauses, answers (A) and (C) are incorrect. Answer (B) is incorrect
because there is no subject for the verb ‘arrived’. Answer (D) is the best answer because there is a
subject, student, for the verb ‘arrived’ and there is a connector ‘after’, to join the two clauses.

The following chart lists common adverb connectors and the sentence patterns used with them.

ADVERB CLAUSE CONNECTORS

TIME CAUSE CONDITION CONTRAST

after, as, before, because, since if, whether even though,


since, until, though, while
when, while
S V Adverb S V
Connector

Matt felt good because he passed.

Adverb
S V comma S V
Connector
Because he passed , Matt felt good.

PRACTICE 2
Each of the following sentences contains more than one clause. Underline the subjects once and
the verbs twice. Circle the connectors. Then indicate if the sentences are correct (C) or incorrect (I).
1. C After the plane circled the airport, it landed on the main runway
2. I The registration process took many hours since the lines so long.
3. This type of medicine can be helpful; it can also have some bad side effects.
4. The waves were amazingly high when the storm hit the coastal town.
5. We need to get a new car whether is on sale or not.
6. Just as the bread came out of the oven, while a wonderful aroma filled the kitchen.
7. Everyone has spent time unpacking boxes since the family moved into the
new house.
8. Although the area is a desert many plants bloom there in the springtime.
9. The drivers on the freeway drove slowly and carefully while the rain was
falling heavily because they did not want to have an accident.
10. If you plan carefully before you take a trip, will have a much better time because
the small details will not cause problems.

18
The following example shows how this sentence pattern could be tested in structure questions
on the TOEFL Tests

PRACTICE 3
Choose the letter of the word or group of words that best complete the sentence.

1. A spacecraft is freed from friction launched into space.


A. it B. it is C. after is D. after it is

2. with their surroundings, or they hide in crevices for protection.


A. Lobsters B. Lobsters blend
C. Lobsters blending D. Because lobsters blend

3. a ball-and-socket joint, the elbow is a simple hinge joint.


A. While the shoulder B. While the shoulder is
C. The shoulder is D. The shoulder

4. A car has several sections with moving parts, of those parts is essential.
A. good lubrication B. well lubricated
C. and good lubrication D. and well lubricated

5. Bears cannot see well small eyes.


A. bears have B. because having
C. because they have D. because of bears

6. at the Isthmus of Panama, so animals were able to migrate between North and
South America.
A. A land bridge existed B. a land bridge existed
C. A land bridge D. With a land bridge

7. mostly made of granite, it also contains some human-made materials.


A. The Empire State Building B. The Empire State Building is
C. Although the Empire State Building is D. Although the Empire State Building is built

8. Pressure differences make the eardrum vibrate the ear.


A. enters the sound waves B. as sound waves
C. sound waves enter D. as sound waves enter

9. An optical microscope magnifies as much as 2,000 times, but art electron microscope _________
as much as a million times.
A. magnifying B. it magnifies
C. can magnify D. magnify it

10. If scientific estimates are accurate, with the Earth about 20,000 years ago.
A. the Canon Diablo meteorite collided
B. the collision of the Canon Diablo meteorite
C. the Canon Diablo meteorite colliding
D. colliding the Canon Diablo meteorite

19
B. READING FOR REFERENCE AND VOCABULARY

Reference questions ask what certain reference words, such as “they” or “this,” refer to. The incorrect
answers are other nouns that are mentioned in the passage. To answer a reference question, substitute
the four choices given to you for the reference word. The one that is the best substitute for it is the
correct answer.

QUESTIONS USUALLY APPEAR ON READING FOR DETAILS AND VOCABULARY


 The word ‘it/they’ in line refers to?
 What does “it/they” in line refer to?
 In line , the word “ ” could be best replaced with?

HOW TO ANSWER THE QUESTION?


 When answering reference questions be aware that the noun closest to the reference word
may not always be the correct answer.
 Reference words may refer to a noun or to a noun phrase made up of several words.
 If you do not know which of the four choices is the correct answer to a reference question,
substitute the choices for the reference word.
 If you are unable to answer a vocabulary-in-context question, try to guess the meaning
from the context by looking for clues.
 Sometimes the meaning of the word is given near the word in the form of a synonym or
paraphrase.
 Sometimes clues are not given but are implied. You can guess the meaning after you have
read the whole passage.
 The answer choices for the vocabulary-in-context questions may appear correct because
they share the literal meaning of the word, but not the meaning as used in the passage.
Look for the meaning as it is used in the passage.

Example:

The Passage

Although “lie detectors” are being used by governments, police departments, and
businesses that all want guaranteed ways of detecting the truth, the results are not always
accurate. Lie detectors are properly called emotion detectors, for their aim is to measure
bodily changes that contradict what a person says. The polygraph machine records
5 changes in heart rate, breathing, blood pressure, and the electrical activity of the skin
(galvanic skin response, or GSR). In the first part of the polygraph test, you are elec-
tronically connected to the machine and asked a few neutral questions (“What is your
name?” “Where do you live?”). Your physical reactions serve as the standard (baseline)
for neutral questions (“What is your name?” “Where do you live?”). Your physical
reactions serve as the standard (baseline) for evaluating what comes next.
10 Then you are asked a few critical questions among the neutral ones (“When did you
rob the bank?”). The assumption is that if you are guilty, your body will reveal the truth,
even if you try to deny it. Your heart rate, respiration, and GSR will change abruptly as
you respond to the incriminating questions.

The question

1. In line 11, the word “ones” refers to


A. reactions B. Evaluations C. Questions D. standards

DISCUSSION
The best answer is (C); in the context of the passage, “ones” replaces the word “questions.”

20
2. The word “it” in line 13 refers to
A. the question B. the assumption C. the truth D. your body

DISCUSSION
The best answer is (C); it refers to the truth.

3. In line 9, the word “assumption” could be best replaced with


A. statement B. belief C. faith D. imagining

DISCUSSION
The best answer is (B); Answer (A) is not correct because if it were a statement we do not know
who made it. The three other choices are synonyms for “assumption” but in the context of the
passage only the word “belief” can be logically used.

PRACTICE 4

George Lucas’s Star Wars changed the direction of American film with some of the
most ingenious special effects contrived for movies of its time. Twenty-two months
were spent on the special effects, including the six months needed to design the
equipment and the more than 1,000 story boards for the effects sequences.
05 A special computerized camera, called a Dykstraflex, was designed to give the illu-
sion of real screen movement. This system, controlled by the camera operator, ena-
bled him or her to pan, tilt, and track around the model, always keeping it in focus.
The breakthrough was the camera’s ability to repeat the identical movements from
shot to shot; thus the effects sequences could be built like a music track; layer upon
layer. The illusion was complete: none of the spaceships in Star Wars ever moved
10 only the camera did.

The star-field backdrop in space was made by punching holes in black plexiglass.
More than 75 models were constructed, with astonishing detail work. On the rebel
blockade runner artists built a tiny cockpit, all done to scale. The miniaturized laser
canons were fully motorized to swivel and tilt by remote control. The light sabers
15 were four-sided blades coated with reflective aluminum, attached to a small motor.
When rotated, they created a flashing light later enhanced by animation.

1. The word “some” in line 1 refers to


A. American film B. direction C. movies D. special effects

2. In line 2, the word “contrived” could be best replaced with which of the following?
A. Discovered B. Created C. Performed D. Utilized

3. In line 6, “this system” refers to


A. the creation of an illusion B. screen movement
C. panning grid tilting around a model D. a special computerized camera

4. The word “it” in line 7 refers to the


A. Model B. camera C. focus D. system

5. The word “they” in line 17 refers to the


A. miniaturized laser cannons B. artists
C. four-sided blades D. severity-five models

21
CHAPTER 4
Noun Clause Connectors
Reading Practices

A. STRUCTURE
I. NOUN CLAUSE CONNECTORS

A noun clause is a clause that functions as a noun; because it is used in a sentence as an object of a
verb (if it follows a verb) or an object of a preposition (if it follows a preposition). Study the clauses
and connectors in the following sentences.

I don’t know why he said such things.

NOUN CLAUSE AS OBJECT OF VERB

I am thinking about

NOUN CLAUSE AS OBJECT OF PREPOSITION

In the first example, there are two clauses, I don’t know and he said such things. These two clauses
are joined with the connector why. ‘Why’ changes the clause he said such things into a noun clause
which functions as the object of the verb don’t know.

In the second example, the two clauses I am thinking and he said such things are also joined by
the connector why. Why changes the clause he said such things into a noun clause, which functions
as the object of the preposition about.

The following chart lists the noun clause connectors and the sentence patterns used with them.

NOUN CLAUSE CONNECTORS


what, when, where, why, how
whether, if

that

S V Noun clause connector S V


Sally explained why she did it

PRACTICE 1

Each of the following sentences contains more than one clause. Underline the subjects once
and the verbs twice. Circle the connectors, then indicate if the sentences are correct (C) or
incorrect (I).

1. ___C _ It is unfortunate that the meal is not ready yet.


2. ___I_ She told me when should pick her up.

22
3. The instructor explained where was the computer lab located.
4. We could not believe what he did to us.
5. Do you want to know if it going to rain tomorrow?
6. We never know whether we will get paid or not.
7. This evening you can decide what do you want to do.
8. The manager explained how wanted the work done.
9. The map showed where the party would be held.
10. Can you tell me why was the mail not delivered today?

II. NOUN CLAUSE CONNECTORS/SUBJECTS


In some cases, a noun clause connector is not just a connector; a noun clause connector can
also be the subject of the clause at the same time. Study the clauses and connectors in the fol-
lowing sentences.

I know what happened yesterday.

Noun Clause Connector/Subject

We are thinking about what happened yesterday.

Noun Clause Connector/Subject

In the first example, there are two clauses: I know and what happened yesterday. These two clauses
are joined by the connector what. It is important to understand that in this sentence the
word ‘what’ serves two functions. It is both the subject of the verb happened and the connector that
joins the two clauses.
In the second example, there are two clauses. In the first clause ‘we’ is the subject of ‘are thinking’.
In the second clause ‘what’ is the subject of happened. What also serves as the connector that joins
the two clauses. The noun clause what happened yesterday functions as the object of the preposition
about.

The following chart lists the noun clause connector/subject and the sentence pattern used with them.

NOUN CLAUSE CONNECTORS/SUBJECTS

who what which

S V (+ O) Noun Clause Connector/ S V

Al told me what happened

23
PRACTICE 2

Each of the following sentences contains more than one clause. Underline the subjects once and
the verbs twice. Circle the connectors. Then indicate if the sentences are correct (C) or incorrect
(I).

1. C The teacher heard who answered the question.


2. I I do not understand what it went wrong.

3. Of the three movies, I can’t decide which is the best.


4. She did not remember who in her class.
5. No one is sure what did it happen in front of the building.
6. We found out which was her favorite type of candy.
7. Do you know what caused the plants to die?
8. I am not sure which it is the most important course in the program.

9. We thought about who would be the best vice president.


10. She saw what in the box in the closet.

PRACTICE 3

Choose the letter of the word or group of words that best completes the sentence.

1. Air near the equator a faster west-to-east motion than air farther from the equator.
A. to have B. it has C. has D. having

2. About 4000 B.C., humans discovered that obtained from special rocks called ores.
A. metals could be B. the ability of metallic
C. possibly metallic D. could metals be

3. quickly after an animal dies.


A. In the degradation of DNA B. Degrading DNA
C. DNA degrades D. For DNA to degrade

4. aerodynamic design has contributed major part in reducing resistance to motion.


A. Improved B. It improves
C. Improvement D. They improve

5. The southern part of Florida is much warmer in the winter than the northern part, so more
to the south.
A. flocking tourists B. touring flocks
C. flocks of tourists D. tourists flock

6. The Moon’s gravity pulls water the near side of the Earth toward the Moon, and this is
what tides to occur.
A. the cause B. causes
C. causing D. the cause of

7. they pick up Fragments of rock which become frozen into the base of the ice.
A. Glaciers move B. Glaciers moving
C. They were glaciers D. As glaciers move

8. The tape measure first evolved f r o m used by the Egyptians.


A. the chains measure B. the chains are measured
C. the chains are measuring D. the measuring chains

24
9. A typical Atlantic hurricane starts as a low pressure system near .
A. Africa coasts B. coast to Africa
C. the African C o as t D. Africa has a coast

10. It is not clear whether the subdivisions of the neocortex units.


A. Individual B. are i n d i v i d u a l
C. they are individual D. individually

B. READING

READING PRACTICE 1:

Carbohydrates, which are sugars, are an essential part of a healthy diet. They provide the
main source of energy for the body, and they also function to flavor and sweeten foods.
Carbohydrates range from simple sugars like glucose to complex sugars
such as amylose and amylopectin.
05 Nutritionists estimate that carbohydrates should make up about one-fourth to one-fifth of a
person’s diet. This translates to about 75-100 grams of carbohydrates per day. A diet that
is deficient in carbohydrates can have an adverse effect on a person’s health. When
the body lacks a sufficient amount of carbohydrates, it must then use its protein sup- plies
for energy, a process called gluconeogenesis. However, this results in a lack of necessary
protein, and further health difficulties may occur. A lack of carbohydrates can
10 also lead to ketosis, a build-up of ketones in the body that causes fatigue, lethargy, and
bad breath.

1. What is the main idea of this passage?


A. Carbohydrates are needed for good health.
B. Carbohydrates prevent a build-up of prot-2ins.
C. Carbohydrates can lead to ketosis.
D. Carbohydrates are an expendable part of a good diet.

2. The word “function” as used in line 2 refers to which of the following?


A. Neglect B. Serve C. Dissolve D. Profess

3. The word “range” as used in line 3 is closest in meaning to which of the following?
A. Probe B. Proceed C. Hail D. Extend

4. According to the passage, what do most nutritionists suggest?


A. Sufficient carbohydrates will prevent gluconeogenesis.
B. Carbohydrates are simple sugars called glucose.
C. Carbohydrates should make up about a quarter of a person’s daily diet.
D. Carbohydrates should be eaten in very small quantities.

5. Which of the following do carbohydrates NOT do?


A. Prevent ketosis B. Cause gluconeogenesis
C. Provide energy for the body D. Flavor and sweeten food

25
6. Which of the following words could best replace “deficient” as used in line 7 ?
A. Outstanding B. Abundant C. Insufficient D. Unequal

7. What does the word “this” refer to in line 9?


A. Using protein supplies for energy C. Having a deficiency in carbohydrates
B. Converting carbohydrates to energy D. Having an insufficient amount of protein

8. According to the passage, which of the following does NOT describe carbohydrates?
A. protein supply C. range of sugars
B. necessity D. An energy source

9. The word “lack” in line 9 is most similar to which of the following?


A. Plethora C. Shortage
B. Derivation D. Commission

10. Which of the following best describes the author’s tone?


A. Sensitive B. Emotional C. Informative D. Regretful

11. Which of the following best describes the organization of this passage’?
A. Cause and result C. Comparison and contrast
B. Specific to general D. Definition and example

READING PRACTICE 2:

The railroad proved to be most viable solution to the great American problem of
distance. Rail transportation was fast, reliable, relatively cheap to construct, and
useable even in winter. Able to go almost anywhere, even over the All gheny
barrier, the railroad defied terrain and weather. Early experiments with
05 railroads involved the use of various kinds of power, including wind, dogs, hors-
es, and finally steam. The first important line was begun by the Baltimore and Ohio
Company significantly on Independence Day in 1828. At the colorful dedication
ceremony, the first stone was laid in Baltimore by Charles Carroll, then age ninety,
the only surviving signer of the Declaration of Independence. But the
10 steam locomotive for railroads, truly a declaration of independence from
primitive transportation, was not, as commonly supposed, a Yankee invention. It
had already been used to a limited extent in England.

1. Early power experiments with railroads included all of the following EXCEPT
A. Steam B. Dogs C. Coal D. horses

2. The dedication ceremony for the first line of the Baltimore and Ohio Company
A. took place through the Allegheny barrier
B. coincided with America’s one hundredth Independence Day
C. included survivors of the signing of the Declaration of Independence
D. occurred in Baltimore

3. It is commonly believed that


A. railroads were unreliable in difficult terrain
B. Americans invented the steam locomotive
C. dogs and horses were necessary for railroad movement
D. the steamship solved America’s distance problems

26
4. All of the following can be said about the railroad EXCEPT that it was
A. quick B. relatively cheap to build
C. primitive D. dependable
5. An appropriate title for this passage would be ______
A. “The Life and Contribution of Charles Carroll”
B. “The Initiation and Impact of the American Railroad”
C. “Railroads in America and England”
D. “The American Distance Problem”

6. In the next to last line, “it” refers to


A. the declaration of independence B. primitive transportation
C. the s t eam locomotive D. a Yankee invention

READING PRACTICE 3:

In 1920, after some thirty-nine years of problems with disease, high costs, and
politics, the Panama Canal was officially opened, finally linking the Atlantic and
Pacific Oceans by allowing ships to pass through the fifty-mile canal zone in- stead
of traveling some seven thousand miles around Cape Horn. It takes a ship
approximately eight hours to complete the trip through the canal and costs an
average of fifteen thousand dollars, one tenth of what it would cost an average ship
to round the Horn. More than fifteen thousand ships pass through its locks each
year.

The French initiated the project but sold their rights to the United States, which
actually began the construction of the project. The latter will control it until the
end of the twentieth century when Panama takes over its duties.

1. Who currently controls the Panama Canal?


A. France B. United States C. Panama D. Canal Zone

2. The word “locks” in line 6 is closest in meaning to


A. securities B. latches C. lakes D. canal gates

3. On the average, how much would it cost a ship to travel around Cape Horn?
A. $1,500 B.$15,000 C.$150,000 D. $1,500,000

4. In what year was construction begun on the canal?


A. 1881 B.1920 C.1939 D.1999

5. It can be inferred from this passage that


A. the canal is a costly project which should be reevaluated
B. despite all the problems involved, the project is beneficial
C. many captains prefer to sail around Cape Horn because it is less expensive
D. problems have made it necessary for three governments to control the canal over the
years

6. In line 2, the word “linking” is closest in meaning to


A. controlling B. dispersing C. detaching D. joining

27
7. In line 7, “initiated” is nearest in meaning to
A. Purchased B. Launched C. forfeited D. forced

8. All of the following are true EXCEPT


A. it costs so much to pass through the locks because very few ships use them
B. the U ni t ed States received the rights to the canal from the French
C. a ship can pass through the canal in only eight hours
D. passing through the canal saves thousands of miles of travel t i m e around Cape Horn

28
CHAPTER 5

Adjective Clause Connectors


Reading Practices

A. STRUCTURE

I. ADJECTIVE CLAUSE CONNECTORS

An adjective clause describes a noun. Because the clause is an adjective, it is positioned directly
after the noun that it describes.

This is the house that I want to buy.

Adjective clause

The house that I want to buy is quiet expensive.

Adjective clause

In the first example, there are two clauses: ‘this’ is the subject of the verb ‘is’, and ‘I’ is the sub-
ject of the verb ‘want’. ‘That’ is the adjective clause connector that joins these two clauses, and
the adjective clause ‘that I want to buy’ describes the noun ‘house’.

In the second example, there are also, two clauses: ‘house’ is the subject of the verb ‘is’, and ‘I’
is the subject of the verb ‘want’. In this sentence also, ‘that’ is the adjective clause connect- or
that joins these two clauses, and the adjective clause ‘that I want to buy’ describes the
noun ‘house’.

The following chart lists the adjective clause connectors and the sentence patterns used with
them.

29
ADJECTIVE CLAUSE CONNECTORS

WHOM WHICH THAT


(for people) (for things) (for people or
things)
S V Adjective S V
clause connect-
or
I like the dress that you are wearing

S Adjective S V V
clause connect-
or
The dress that you are wearing is beautiful

NOTE:

The adjective connectors can be omitted. This omission is very common in spoken English
or in casual written English. It is not as common in formal English or in structure
questions on the TOEFL test.

PRACTICE 1

Each of the following sentences contains more than one clause. Underline the subjects once and
the verbs twice. Circle the connectors. Then indicate if the sentences are correct (C) or incorrect
(I).
1. C I did not believe the story that he told me.
2. I Ms. Brown, whom did you recommend for the job, will start work tomorrow.
3. The lecture notes which lent me were not clearly written.
4. Sally has an appointment with the hairdresser whom you recommended.
5. The phone number that you gave me.
6. She is able to solve all the problems which did she cause.
7. The day that she spent on the beach left her sunburned.
8. Next week I am going to visit my cousins, whom have not seen in several
years.
9. Did you forget the promise whom made?
10. The teacher whom the students like the most is their history teacher.

II. ADJECTIVE CLAUSE CONNECTOR/SUBJECTS

Adjective clause connectors can be used to introduce clauses that describe nouns. We will see that
in some cases an adjective clause connector is not just a connector; an adjective clause connector
can also be the subject of the clause at the same time.

30
We are looking at a house that is quite expensive.

Adjective clause

Adjective clause connector/Subject

The house that is quite expensive seems like a great house.

Adjective clause

Adjective clause connector/Subject

In the first example, there are two clauses: ‘we’ is the subject of the verb ‘are looking’, and ‘that’
is the subject of the verb ‘is’. These two clauses are joined with the connector ‘that’. Notice that in
this example the word ‘that’ serves two functions at the same time: it is the subject of the verb is,
and it is the connector that joins the two clauses. The adjective clause that is quite expensive describes
the noun house.

In the second example, there are also two clauses: house is the subject of the verb seems, and that
is the subject of the verb is. In this example, that also serves two functions: it is the subject of the
verb is, and it is the connector that joins the two clauses. Because that is quite expensive is an
adjective clause describing the noun house, it directly follows house. The following chart lists the
adjective clause connector/subjects and the sentence patterns used with them.

ADJE TIVE CLAUSE CONNECTORS / SUBJECTS

WHOM WHICH THAT


(for people) (for things) (for people or
things)
S V Adj. Clause Con- V
nector/S

I bought the dress that was on sale

S Adj. Clause Con- V V


nector/S

The dress that was on sale was beautiful

NOTE: Although adjective clause connectors can be omitted in informal English,


adjective clause connector/subjects can never be omitted.

31
PRACTICE 2

Each of the following sentences contains more than one clause. Underline the subjects once and the
verbs twice. Circle the connectors. Then indicate if the sentences are correct (C) or incorrect (I).
1. C The children that were vaccinated did not get sick.
2. I I did not vote for the politician who he just won the election.
3. The dog that barking belongs to my neighbor.
4. I took two of the blue pills, which were very effective.
5. We rented an apartment from the landlord who does he own the buildings on
Maple Street.
6. She forgot to attend the meeting which it began at 11:00.
7. Any student who does not turn in the paper by Friday will fail the class.
8. The people which came in late had to sit at the back.
9. The courses that satisfy the graduation requirements they are difficult.
10. After dinner she went to visit her parents, who were living down the street.

PRACTICE 3
Choose the letter of the word or group of words that best completes the sentence.

1. is famous as the home of the United States Naval Academy.


A. Annapolis B. Because of Annapolis C. Why Annapolis D. Because Annapolis

2. Some scientists think be a planet but a moon of Neptune.


A. that Pluto does not seem B. not Pluto
C. Pluto that might not D. that Pluto that might not

3. With of sophisticated oil lamps, elaborate tools were made to cut the wicks.
A. Appeared B. the appearance C. the appearance was D. it appeared
4. Fort Union was the site of what principal fur-trading past on the upper Missouri
River.

A. the B. being the C. was the D. it was the

5. Since commercial risk, it has to appeal to a large audience to justify its cost.
A. the face of the movie B. moving faces C. a movie faces D. to face a movie

6. A current of water known as the Gulf Stream comes up from the Gulf of Mexico, and then______
the North Atlantic toward Europe.
A. it crosses B. crossing C. with its crosses D. crosses it

7. Systems the two symbols 0 and 1 are called binary number systems.
A. use B. they use C. uses D. using

8. Genes, the blueprints for cell construction, exist in tightly organized packages
called chromosomes.
A. are B. they are C. which D. which are

9. The Earth’s atmosphere consists of gases in place around the Earth by the gravita
tional pull of the planet.
A. held B. hold C. it holds D. the hold
10. Oscar Hammerstein II collaborated with a number of composers including Jerome Kern, whom
in writing the musical Show Boat.
A. joined B. was joined C. he joined D. joining

32
B. READING

READING PRACTICE 1:
For hundreds of years in the early history of America, pirates sailed through coastal
waters, Pillaging and plundering all in their path. They stole from other ships and
stole from coastal towns; not content only to steal, they destroyed everything they
could not carry away. Some of the pirate ships amassed large treasures, the fates
of which are unknown, leaving people of today to wonder at their whereabouts and
5 to dream of one day coming across some lost treasure.

One notoriously large treasure was on the pirate ship Whidah, which sank in the wa-
ters off Cape Cod during a strong storm in 1717. A hundred of the crew members
went down with the ship, along with its treasure of coins, gold, silver, and jewels.
The treasure on board had an estimated value, on today’s market, of more than 100
million dollars. The remains of the Whidah were discovered in 1984 by Barry
10 Clifford, who had spent years of painstaking research and tireless searching, only
finally, to locate the ship about 500 yards from shore. A considerable amount of
treasure from the centuries-old ship has been recovered from its watery grave, but
there isclearly still a lot more out there. Just as a reminder of what the waters off the
coast have been protecting for hundreds of years, occasional pieces of gold, or
15 silver, or jewels still wash up on the beaches, and lucky beach-goers find pieces of
the treasure.

1. The passage mainly discusses


A. early pirates B. a large pirate treasure
C. what really happened to the Whidah’spirates D. why people go to the beach

2. It is NOT mentioned in the passage that pirates did which of the following?
A. They killed lots of people. B. They robbed other ships.
C. They took things from towns. D. They gathered big treasures.

3. The word “amassed” in line 4 is closest in meaning to


A. sold B. hid C. transported D. gathered

4. It is implied in the passage that the Whidah’s crew


A. died B. went diving
C. searched for the treasure D. escaped with parts of the treasure

5. Which of the following is NOT mentioned as part of the treasure of the Whidah?
A. Art objects B. Coins C. Gold and silver D. Jewels

6. The word “estimated” in line 9 is closest in meaning to which of the following?


A. Known B. Sol C. Approximate D. Decided

7. The passage indicates that the cargo of the Whidah is worth about
A. $100,000 B. $1,000,000 C. $10,000,000 D. $100,000,000

8. The work that Barry Clifford did to locate the Whidah was NOT
A. Successful B. Effortless C. Detailed D. Lengthy

9. It is mentioned in the passage that the treasure of the Whidah


A. is not very valuable B. is all in museums
C. has not all been found D. was taken to shore by the pirates

33
10. The paragraph following the passage most likely discusses
A. what Barry Clifford is doing today
B. the fate of the Whidah’s crew
C. Other storms in the area of Cape Cod
D. additional pieces that turn up from the Whidah’s treasure

READING PRACTICE 2:

It is a characteristic of human nature that people like to get together and have fun,
and people living during America’s frontier days were no exception. However,
because life was hard and the necessities of day-to-day living took up their time, it
was common for recreation to be combined with activities necessary for survival.
5 One example of such a form of recreation was logrolling. Many frontier areas were
heavily wooded, and in order to settle an area it was necessary to remove the trees.
A settler could cut down the trees alone, but help was needed to move the
cut trees. After a settler had cut a bunch of trees, he would then invite his neighbors
over for a logrolling. A logrolling was a community event where families got
10 together for a combination of work and fun. The women would bring food and have
a much needed and infrequent opportunity to relax and chat with friends, the
children would play together exuberantly, and the men would hold lively com-
petitions that involved rolling logs from place to place as quickly as possible. This
was a day of fun of everyone involved, but at its foundation was the need to clear the
land.

1. The main idea of the passage is that in America’s frontier days _______
A. people combined work with recreation
B. people cleared land by rolling logs
C. it was necessary for early settlers to clear the land
D. a logrolling involved the community

2. The expression “day-to-day” in line 3 could best be replaced by which of the following?
A. Daytime B. Everyday C. Day after day D. Today’s

3. The word “survival” in line 4 is closest in meaning t o


A. existence B. a lifetime C. physical exercise D. society

4. According to the passage, what did people have to do first to settle an area?
A. Develop recreation i d e a s B. Build farms
C. Get rid of the trees D. Invite neighbors’ over

5. According to the passage, which of the following is NOT true about a logrolling?
A. It involved a lot of people. B. It could be enjoyable.
C. There could be a lot of movement. D. It was rather quiet.

6. According to the passage, what did people have to do first to settle an area?
A. Develop recreation ideas B. Build farms
C. Get rid of the trees D. Invite neighbors’ over

7. According to the passage, which of the following is NOT true about a logrolling?
A. It involved a lot of people. B. It could be enjoyable.
C. There could be a lot of movement. D. It was rather quiet.

34
CHAPTER 6
Agreements
Reading Practices

THE WRITTEN EXPRESSION QUESTIONS

Written expression questions that test your knowledge of the correct way to express yourself in
English writing appear on both the paper TOEFL test and the computer TOEFL test. Each
question consists of one sentence in which four words or groups of words have been under-lined.
You must choose the underlined word or group of words that is not correct. Look at an example
of a written expression question from the paper TOEFL test.

Example:

Both of a halibut’s eyes are on one sides of its head.


A B C D

In this example, you should notice that the singular quantifier one is accompanied by the plural
noun sides. One should be accompanied by the singular noun side. You should choose answer
(C) because answer (C) is not correct.

Now look at another example:

Colorful rock formations is found in Haleakala’s crater.

In this example, you should notice that the singular verb ‘is’ does not agree with the plural
subject formations. The plural verb should be used with a plural sub-
ject. You should choos e the word is to answer this question b e c a u s e
is not correct.

PROCEDURES FOR THE WRITTEN EXPRESSION QUESTIONS

1. First, look at the underlined words or groups of words. You want to see if you can spot which
of the four answer choices is not correct.

2. If you have been unable to find the error by looking only at the four underlined expressions,
then read the complete sentence. Often an underlined expression is incorrect because of
something in another part of the sentence.

Next you should move on to the language skills. The following language skills will help you to
implement the appropriate strategies and procedures in the written expression questions on both
the paper TOEFL. .

35
A. AGREEMENTS

1. SUBJECT— VERB AGREEMENT

Subject/verb agreement is simple: if the subject of a sentence is singular, then the verb must be sin-
gular; if the subject of the sentence is plural, then the verb must be plural. An -s on a verb usually
indicates that a verb is singular, while an -s on a noun usually indicates that the noun is plural. (Do
not forget irregular plurals of nouns, such as women, children, and people.)
Examples: Subject — Verb Agreement

The dog barks at night.


The dogs bark at night.

In the first example, the singular subject ‘dog’ requires a singular verb, ‘barks’. In the second ex-
ample, the plural subject ‘dogs’ requires a plural verb ‘bark’.

2. AGREEMENT AFTER PREPOSITIONAL PHRASE


Sometimes prepositional phrases can come between the subject and the verb on the TOEFL test, and
this can cause confusion. If the object of the preposition is singular and the subject is plural, or if the
object of the preposition is plural and the subject is singular, there can be a problem in making the
subject and verb agree.
Examples: Agreement after prepositional phrases

The door (to the rooms) are* locked.


singular plural

The doors (to the room) is* locked.


plural singular

(* indicates an error)

In the first example, you might think that ‘rooms’ is the subject because it comes directly in front
of the verb ‘are’. However, ‘rooms’ is not the subject because it is the object of the preposition ‘to’.
The subject of the sentence is ‘door’, so the verb should be ‘is’. In the second example, you might
think that ‘room’ is the subject because it comes directly in front of the verb ‘is’. You should
recognize in this example that ‘room’ is not the subject because it is the object of the preposition
‘to’. Because the subject of the sentence is ‘doors’, the verb should be ‘are’.

The following chart outlines the key information that you should understand about subject/verb
Agreement with prepositional phrases.

SUBJECT—VERB AGREEMENT WITH PREPOSITIONAL PHRASES

S (prepositional phrase) V

When a prepositional phrase comes between the subject and the verb, be sure that the verb agrees
with the subject.

36
PRACTICE 1
Each of the following sentences has one or more prepositional phrases between the subject and
verb. Put parentheses around the prepositional phrases. Underline the subjects once and the verbs
twice. Then indicate if the sentences are correct (C) or incorrect (I).
1. The subject (of the lectures) was quite interesting. ( C )
2. The supplies (for the camping trip) needs to be packed. ( I )
3. The chairs under the table in the dining room is quite comfortable. ( )
4. The players on the winning team in the competition put forth a lot of effort. ( )
5. The food for the guests at the party are on the long tables. ( )
6. The cost of the clothes was higher than I had expected. ( )
7. The rugs in the front rooms of the house are going to be washed today. ( )
8. The waiters and waitresses in this restaurant always serves the food efficiently. ( )
9. The lights in the corner of the room need to be kept on all night. ( )
10. The meeting of the members of the council begins at 3:00 in the afternoon. ( )

3. AGREEMENT AFTER EXPRESSIONS OF QUANTITY


A particular agreement problem occurs when the subject is an expression of quantity like all, most,
or some followed by the preposition of. In this situation, the subject (all, most, or some) can be
singular or plural, depending on what follows the preposition of.

Most (of the meal) was delicious.


singular

Most (of the meals) were delicious.


plural

Most (of the food) was delicious.


singular

In the first example, the subject most refers to the singular noun meal, so the correct verb is the
singular verb was.
In the second example, the subject most refers to the plural noun meals, so the correct verb is
the plural verb were.
In the third example, the subject most refers to the uncountable noun food, so the correct verb is
the singular verb was.

These sentences contain examples of the types of problems that are common on the TOEFL test.

All (of the book) were* interesting.

Half (of the students) is* late to class.

In the first example, the plural verb ‘were’ should be the singular verb ‘was’ because the subject all
refers to the singular noun ‘book’.
In the second example, the singular verb ‘is’ should be the plural verb ‘are’ because the subject half
refers to the plural noun ‘students’.

37
The following chart outlines the key information that you should understand about subject/verb
agreement after expressions of quantity.

SUBJECT/VERB AGREEMENT
AFTER EXPRESSIONS OF QUANTITY

All
Most
Some OF THE (OBJECT) V
Half
Part

When an expression of quantity using of is the subject, the verb agrees with the object.

PRACTICE 2
Each of the following sentences has a quantity expression as the subject. Underline the sub-
jects once and the verbs twice. Circle the objects that the verbs agree with. Then indicate if the
sentences are correct (C) or incorrect (I).

1. Half of the students in the class arrive early. (C)


2. Some of the fruit are rotten. (I)
3. All of the next chapter contains very important information. ( )
4. Most of the people in the room is paying attention. ( )
5. Part of the soup is left on the stove. ( )
6. Some of the movie were just too violent for me. ( )
7. All of the details in the report needs to be checked. ( )
8. Most of the money is needed to pay the bills. ( )
9. The first half of the class consists of lecture and note-taking. ( )
10. Some of the questions on the test was impossible to answer. ( )

4. AGREEMENT AFTER CERTAIN WORDS

Certain words in English are always grammatically singular, even though they might have plural
meanings.

Everybody in the theater are watching* the film attentively.

Even though we understand from this example that a lot of people are watching the film, every-
body is singular and requires a singular verb. The plural verb are watching should be changed to
the singular verb is watching.

38
The following chart lists the grammatically singular words that have plural meanings.

SUBJECT/VERB AGREEMENT AFTER CERTAIN WORDS

These words or expressions are grammatically singular, so they take singular verbs:

anybody everybody nobody somebody each (+noun)


anyone everyone no one someone every (+noun)
anything everything nothing something

PRACTICE 3

Each of the following sentences contains one of the words that are grammatically singular but
have a plural meaning. Underline these words once and underline the verbs twice. Then indicate
if the sentences are correct (C) or incorrect (I).

1. Anybody are welcome at the party. (I)


2. No one here is afraid of skydiving. (C)
3. Everyone in the world needs love and respect. (_____ )
4. Someone have to clean up the house. ( )
5. Each plant in the garden appear healthy and s t r o n g . ( )
6. You should understand that anything i s possible. ( )
7. Everything in the salad are good for you. ( )
8. Nobody in the class have completed the assignment o n time. ( )
9. I am sure that every detail have been considered. ( )
10. Everybody know the rules, but somebody is not following t h e m . ( )

PRACTICE 4
Each of the following sentences may have a problem with subject/verb agreement. Underline the
subjects once and the verbs twice. Then indicate if the sentences are correct (C) or incorrect (I).

1. The receptionist in the entry way to the offices is able to answer your questions. ( )
2. All of the information in the documents are important. ( )
3. Anyone in one of the classes has LO, take the final exam. ( )
4. The coordinator of community services are arranging the program. ( )
5. Most of the car are covered with mud. ( )
6. Nothing more is going to be completed today. ( )
7. The drinks in the pitchers on the table in the ballroom is for everyone.
8. Everybody were told to be here at 8:00, but somebody is not here. ( )
9. Some of the meetings at the conference are limited to ten participants. ( )
10. The sauce on the vegetables in the yellow bowl taste really delicious. ( )

PRACTICE 5
Choose the letter of the underlined word or group of words that is not correct.

1. Nobody know when the process of glass-making was invented.


A B C D

39
2. Sugars like glucose is made up of carbon, hydrogen, and oxygen atoms.
A B C D

3. Part of the electricity used in the United States today come from hydroelectric sources.
A B C D

4. The languages of the world presents a vast array of structural similarities and differences.
A B C D

5. The rise of multinationals have resulted in a great deal of legal ambiguity because multinationals
A B
can operate in so many jurisdictions.
C D

6. All of the east–west interstate highways in the United States has even numbers, while north–
A B
south interstate highways are odd-numbered.
C D

7. When a massive star in the large Magellanic Cloud exploded in 1987, a wave of neutrino were
A B C
detected on earth.
D

8. Some of the agricultural practices used today is responsible for fostering desertification.
A B C D

9. Every open space in the targeted area that has grass and a few bushes are occupied by the white
A B C D
-crowned sparrow.

10. Krakatoa is remembered as the volcano that put so much ash into the air that sunsets around
A B
the world was affected for two years afterward.
C D

PRACTICE 6
Choose the letter of the word or group of words that best completes the sentence.

1. the earliest system of writing.


A. The constitution of pictograms B. Pictograms in the constitution
C. Constitute the pictograms D. Pictograms constitute

2. At temperatures absolute zero, substances possess minimal energy.


A. approach B. approaches
C. approaching D. they approach

3. The Earth’s one-year revolution around the Sun changes how on one hemisphere or
the other.
A. falling sunlight B. the fall of sunlight
C. sunlight in the fall D. sunlight falls

40
4. Though sporadic interest in regional dialects for centuries, the first large scale
systematic studies did not take place until the nineteenth century.
A. has existed B. it existed C. has it existed D. existing with it

Choose the letter of the underlined word or group of words that is not correct.

5. The waters of the Chattahoochee River fills Lake Lanier.


A B C D

6. The first set of false teeth similar to those in use today it was made in France in the 1780s.
A B C D

7. The term “Yankee” was originally a nickname for people from New England, but now any
A
one from the United States are referred to as a Yankee.
B C D

8. A network of small arteries, mostly sandwiched between the skin and the underlying muscles,
A B
supply blood to the face and scalp.
C D

9. Mosquito is a small tree in the Southwest who can withstand the severest drought.
A B C D

10. At the end of the Revolution, most of the army units of the young nation was almost entirely
A B
disbanded, leaving a total national military force of eighty men in 1784.
C D

The Indian tribes of eastern North America were among the first Indians to meet English settlers.
In 1621, the Indians and English celebrated a good harvest and peace together in Plymouth colony.
But the good relationship did not last long. The Indians were slow to perceive that their way of life
was incompatible with that of the English. They often sold their land or gave it away with- out
realizing that it would no longer be theirs. They used the land mainly for hunting and were willing
to let the English hunt on it with them. But the English cut the trees, drove out the game, and
evicted the Indians. Before the Indians realized what was happening, they were outnumbered.

1. According to the passage, the major downfall of the Indians was


A. being slow to understand their incompatibility with the English
B. selling or giving away their land
C. becoming outnumbered
D. letting the English hunt on their land

2. The English did all of the following EXCEPT


A. cut down trees
B. drive out game
C. evict the Indians
D. use the land mainly for hunting

41
3. The Indians at one time did all of the following EXCEPT
A. live in eastern North America
B. give away their land
C. evict the English
D. sell their land

4. According to the passage, the English


A. never paid for Indian land
B. hunted without the Indian
C. outnumbered the Indians
D. eventually had a lifestyle similar to that of the Indians

42
CHAPTER 7
Parallel Structures
Reading: Inference Questions

A. PARALLEL STRUCTURES

1. PARALLEL STRUCTURE WITH COORDINATE CONJUNCTIONS

The job of the coordinate conjunctions (and, but, or) is to join together equal expressions. In other
words, what is on one side of these words must be parallel to what is on the other side. These con-
junctions can join nouns, or verbs, or adjectives, or phrases, or clauses; they just must join together
the same structures.

Here are examples of equal structures correctly joined by coordinate conjunctions:

1. He discussed the problem with the nurse and the doctor.


2. The professor was knowledgeable but boring.
3. She hikes, jogs, or rides her bicycle whenever she can.
4. There are meetings in the morning, in the afternoon, and in the evening.
5. You can do the work because you want to do it or because you have to do it.

In the first example, two nouns are joined by the coordinate conjunction and.
In the second example, two adjectives are joined by the coordinate conjunction but.
In the third example, three verbs are joined by the coordinate conjunction or.
In the fourth example, three phrases are joined by the coordinate conjunction and.
In the last example, two clauses are joined by the coordinate conjunction or.

The sentences that follow show the types of errors in parallel structure that are common on the-
TOEFL test.

The evening dress was beauty* but expensive.


The student reads each chapter, takes a lot of notes, and memories* the material.

In the first example, the coordinate conjunction ‘but’ joins the noun ‘beauty’ and the adjective
‘expensive’. The adjective ‘beautiful’ is needed in place of ‘beauty’.

In the second example, the coordinate conjunction ‘and’ joins the verb ‘reads’, the verb ‘takes’,
and the plural noun ‘memories’. The verb ‘memorizes’ is needed in place of memories.

The following chart outlines the use of parallel structures with coordinate conjunction .

43
PARALLEL STRUCTURE
WITH COORDINATE CONJUNCTIONS

and
(same structure) but
or

and
(same structure), (same structure), but (same structure)
or

PRACTICE 1

Each of the following sentences contains words or groups of words that should be parallel. Circle
the word that indicates that the sentence should have parallel parts. Underline the parts that should
be parallel. Then indicate if the sentences are correct (C) or incorrect (I).

1. The pastries in that shop are very expensive but quite deliciously. (I)
2. You can find some change to buy a paper in the drawer, on top of the dresser, or in the jar.
(C)
3. The living room was decorated with expensive paintings and elegance amps. ( )
4. He knew that the financial problems were serious, that the situation was not going to
improve, and that he needed to get a job. ( )
5. All day long during the trip to the mountains, they were skiing, sledding, or played in the
snow. ( )
6. The car needs new tires but not a new engine. ( )
7. He stops working when he gets too tired to continue or when he has finished. ( )
8. To get to the office, you should go through this door, turn to the left, and continuation down
the hall. ( )
9. For dessert we could serve lemon pie, fruit tarts, chocolate cake, or butter cookies. ( )
10. The sick child needs some medicine, some juice, and to rest. ( )

2. PARALLEL STRUCTURE WITH PAIRED CONJUNCTIONS

The paired conjunctions both … and, either ... or, neither ... nor, and not only but also require
parallel structures.

The lecture was both informative and enjoyable.


Either the history exam or the physics exam is on Tuesday.
The missing papers are neither on his desk nor in the file.
He visited not only his cousin but also his grandmother.

The following is not parallel and must be corrected:


They want either to play tennis or golf*.

It is not correct because ‘to play’ tennis is not parallel to ‘golf’. It can be corrected in different
ways.

44
They want either to play tennis or to play golf.
They want to play either tennis or golf.

When you are using these paired conjunctions, be sure that the correct parts are used together.
The following are incorrect:
He lent me both some paper or* a pencil.
Either breakfast nor* lunch is included in the price.

These sentences are incorrect because the wrong parts of the paired conjunctions are used
together. In the first example, and should be used with both. In the second example, or should be
used with either.
The following chart outlines the use of parallel structure with paired conjunctions.

PARALLEL STRUCTURE
WITH PAIRED CONJUNCTIONS
both And
either or
(same structure) nor (same structure)
neither
not only but also

PRACTICE 2

Each of the following sentences contains words or groups of words that should be parallel. Circle
the words that indicate that the sentence should have parallel parts. Underline the parts that should
be parallel. Then indicate if the sentences are correct (C) or incorrect (I).

1. He either lied or telling an unbelievable story. ( I )


2. The music at the concert was neither well played nor well liked. ( C )
3. He regularly studies both in the morning or in the evening. ( ____ )
4. The play that we saw last night was not only rather delightful but also quite meaningful.
( )
5. He married her neither for her ability to cook nor her ability to clean house. ( )
6. The discussion was both exciting and interest. ( )
7. He withdrew all the money not only from the checking account but also from the savings
account. ( )
8. Neither the teacher or the students are ready to leave the classroom. ( )
9. You can meet with me either in the next few minutes or at 4:00. ( )
10. John is an adventurous person who enjoys not only skydiving but also goes parasailing.
( )

45
PRACTICE 3

Each of the following sentences contains words or groups of words that should be parallel. Circle
the word or words that indicate that the sentence should have parallel parts. Underline the parts
that should be parallel. Then indicate if the sentences are correct (C) or incorrect (I).
1. The advertisements appeared in the newspaper and on the radio. ( )
2. She is trained as both an accountant and in nursing. ( )
3. We can take either my car or yours to the party. ( )
4. The coffee is too hot, too bitter, and too strength. ( )
5. He not only passed the test but also receiving the highest score in the class. ( )
6. Your ideas are neither more important or less important than the ideas of the others. ( )
7. The meeting lasted only an hour but still seeming too long. ( )
8. The novel was both emotional and description. ( __)
9. Either the counselor or her secretary can help you with that problem. ( )
10. The leaves from the tree fell in the yard, in the pool, the driveway, and on the sidewalk.
( )

PRACTICE 4
Choose the letter of the underlined word or group of words that is not correct.

1. Ballpoint pens are less versatile but more population than fountain pens.
A B C D

2. Riddles vary greatly in both grammatical and phonology form.


A B C D

3. Blood pressure is measured by feeling the pulse and apply a force to the arm.
A B C D

4. The Moon has no atmosphere, no air, and no watery.


A B C D

5. The first matches were too hard to ignite, a mess, or too dangerously easy to ignite.
A B C D

6. A 1971 U.S. government policy not only put warnings on cigarette packs but also banning
A B C
television advertising of cigarettes.
D

7. Demand, beauty, durability, rare, and perfection of cutting determine the value of
A B C D
a gemstone.

8. The Harvard Yard, which was Harvard’s original campus, is still a major at traction
A B C

for both students and visiting.


D

46
9. In 1862, the American Confederacy raised the Merrimack, renamed it Virginia, covered it
A B
with iron plates, and an outfit it with ten guns.
C D

10. The liquid crystals in a liquid crystal display (LCD) affect the polarized light so that it is
A B C
either blocked and reflected by the segments of the display.
D

PRACTICE 5
Choose the letter of the word or group of words that best completes the sentence.

1. Most cells in multi celled organisms perform functions.


A. specialize B. specialized

C. they specialize D. specialization

2. The big island of Hawaii, in the middle of the Pacific Ocean, by five volcanoes.
A. creation B. it was created
C. creating D. was created

3. The Sun uses up over four million tons of hydrogen per second, still has enough
hydrogen to last for the next five billion years.
A. it does not B. it C. but it D. to it

4. For Katherine L. Bates, who the top of Pikes Peak in 1893, the view provided the
inspiration for her hymn “America the Beautiful.”
A. reached B. she reached C. reaching D. she was reaching

Choose the letter of the underlined word or group of words that is not correct.

5. Coal, petroleum, and natural gaseous are all fossil fuels.


A B C D

6. The mass of neutron stars generally range from one-tenth to twice the mass of the Sun.
A B C D

7. Grasses grow in ways that help them to survive being nibbled, chilly, or dried.
A B C D

8. Most of Hemingway’s novels glorifies heroic exploits such as bullfighting or boxing.


A B C D

9. Paleographers study ancient and medieval handwriting in order to establish not only its age and
A B C D
also its background.

10. The sounds produced by bullfrogs and toads vary greatly because each species have its own
A B C
particular call.
D

47
B. READING
INFERENCE QUESTIONS
Inference questions are perhaps the most difficult questions to answer in the reading comprehension
section. The answers to these questions are not directly stated in the passage but are understood, or
implied.

The following are examples of inference questions:

 Which of the following can be inferred about?


 Which of the following can be inferred from the passage?
 From the passage, it can be inferred that ....
 The passage implies that ....
 The author implies that ....
 The passage suggests that ....
 It is most probable that ....

To answer inference questions, you must decide what logical conclusion follows from the facts
stated in the passage. These ideas may be what the author believes to be true but has not stated
in the passage.

EXAMPLE

Lie detectors are properly called emotion detectors, for their aim is to measure bodily
changes that contradict what a person says.

1. What can be inferred from the sentence?

 Lie detectors record a person’s emotions.


 Emotions can contradict what a person says.

2. What cannot be inferred from the sentence?

 People always say what they are feeling. (Bodily changes can contradict what a per-
son says.)
 Lie detectors cause changes in emotions. (No. Lie detectors measure or record bodily
changes.)

READING EXAMPLE

Although “lie detectors” are being used by governments, police departments, and busi-
nesses that all want guaranteed ways of detecting the truth, the results are not always
accurate. Lie detectors are properly called emotion detectors, for their aim is to measure
bodily changes that contradict what a person says. The polygraph machine records
changes in heart rate, breathing, blood pressure, and the electrical activity of the skin
(galvanic skin response or GSR). In the first part of the polygraph test, you are electron-
ically connected to the machine and asked a few neutral questions (“What is your name?”
“Where do, you live?”). Your physical reactions serve as the standard (baseline) for
evaluating what comes next.

48
Then you are asked a few critical questions among the neutral ones (“’When did you rob
the bank?). The assumption is that if you are guilty, your body will reveal the truth, even
if you try to deny it. Your heart rate, respiration, and GSR will change abruptly as you
respond to the incriminating questions. That is the theory; but psychologists have found
that lie detectors are simply not reliable. Since most physical changes are the same across
the emotions, machines cannot tell whether you are feeling guilty, angry, nervous, thrilled,
or revved up from an exciting day. Innocent people may be tense and nervous about the
whole procedure. They may react physiologically to a certain word (“bank”) not because
they robbed it, but because they recently bounced a check. In either case the machine will
record a “lie.” The reverse mistake is also common. Some practiced liars can lie without
flinching, and others learn to beat the machine by tensing muscles or thinking about an
exciting experience during neutral questions.

QUESTION
1. Which of the following can be inferred from the passage?
A. Lie detectors are very reliable.
B. Innocent people are never found guilty.
C. Psychologists never argue about anything.
D. Most people cannot control their bodily reactions.

ANSWER

Answer (A) is incorrect because the second paragraph states that psychologists find lie detec-
tors to be unreliable.
Answer (B) is also incorrect, because the passage states that the lie detector may record inno-
cent people as lying in response to a question, not because they are guilty but because they are
nervous. Therefore, innocent people may be found guilty.
Answer (C) is also incorrect, because although psychologists may agree on the unreliability of
lie detectors, we do not know whether they agree on other issues. Therefore, this cannot be
inferred.
Answer (D) is the best answer. Although we know that some practiced liars can control their
bodily reactions, in general, we can infer that most people cannot control their bodily reac- tions
such as breathing rate, heart rate, blood pressure, and the electrical activity of the skin.

QUESTION
2. From the passage, it can be inferred that
A. Emotions are all the same.
B. Psychologists would not want the detectors used to prove someone’s guilt.
C. Neutral questions reveal the truth.
D. Psychologists are unreliable.

ANSWER

Answer (A) cannot be inferred. The passage states that physical changes are the same across
all emotions, but that does not mean that emotions are all the same.
Answer (C) also cannot be inferred since the passage states that some practiced liars can beat
the machine
Answer (D) cannot be inferred since the passage states that psychologists find the detectors
unreliable; there is no indication in the passage that psychologists are unreliable.
The best answer is (B) since it can be inferred that psychologists would not want to use lie
detectors to prove someone’s guilt, because they consider them to be unreliable.

49
STRATEGIES

 Go beyond the information stated in the passage.


 Draw a conclusion or reason out what is implied — that is, what the author of the passage
means or believes to be true but has not stated in the passage.
 Remember that the answer to the question will not be stated in words in the passage.
 Beware of answer choices that go beyond what you can logically infer from the passage.
Wrong answer choices will often be too exaggerated or overstated to be precisely correct.

PRACTICE
A major revolution for the automated office is electronic mail. The customary postal sys-
tem requires messages written on paper to be transmitted physically from one location to
another. With electronic mail, messages are converted into electronic signals, transmitted
anywhere in the world, and then changed back into the original written form, all in sever-
5 al seconds or minutes at most.

Through the use of video screens in company offices, a single document can be transmitted
to hundreds of people in dozens of branch offices at the same time. Thus, electronic mail,
along with databases, can be an important asset in teleconferences. Furthermore, the use of
electronic mail in the form of a “mailbox” attached to a telephone is also of great value.
10 Since in offices many telephone calls go uncompleted on the first attempt, with electronic
mail, two-way conversion is not essential, so it reduces telephone use and saves time.
Electronic mail is far more expensive than the postal system, as it must compete for trans-
mission space on satellite, telephone, and cable links. But planned increases in satellite
communications should lower the price and in the spread of electronic mail.

1. It can be inferred from the passage that the advantages of electronic mail are
A. its low cost and efficiency.
B. its use of the postal system and telephone “ mailboxes”.
C. its role in satellite communications and teleconferencing
D. its speed and utility

2. The passage implies that electronic mail


A. will be used more often in the future
B. will soon outlive its usefulness
C. is too costly for most offices
D. Planned increases in satellite communications should lower

3. Which of the following can be inferred from the passage?


A. Electronic mail saves money but not time.
B. Electronic mail has more than one use in the office setting.
C. Electronic mail will eventually replace the telephone.
D. Electronic mail is not a new phenomenon.

50
CHAPTER 8
Participles after Have and Be, Base Form Verbs after Modals
Reading: ‘Where’ Questions

A. PARTICIPLES AFTER HAVE AND BE, BASE FORM VERBS AFTER MODALS

1. PAST PARTICIPLES AFTER HAVE

Whenever you see the verb have in any of its forms (have, has, having, had), be sure that the
verb that follows it is in the past participle form.

We had complete* the work.


They have went* to the market.

In the first example, ‘complete’ should be the past participle ‘completed’ because it is after had.
In the second example, ‘went’ should be the past participle ‘gone’ because it is after have.

The following chart outlines the use of verb forms after have.

VERB FORMS AFTER HAVE


HAVE + past participle

PRACTICE 1

Each of the following sentences contains a verb formed with have. Underline the verbs twice.
Then indicate if the sentences are correct (C) or incorrect (I).

1. We have already hearing the good news. (I)


2. She has ridden her bicycle to school every day. ( C )
3. I have always believed you. ( )
4. He has find the missing car keys. ( )
5. They have put their money in a savings account. ( )
6. Their parents have allowed them to stay up late. ( )
7. She has never ran away from home before. ( )
8. Have you ever saw a ghost? ( )
9. They have taken three tests already this week. ( )
10. He has offer me a high-paying job. ( )

2. PRESENT PARTICIPLES OR PAST PARTICIPLES AFTER BE

The verb be in any of its forms (am, is, are, was, were, be, been, being) can be followed by an-
other verb. This verb should be in either the ‘present participle’ or the ‘past participle’ form.

51
They are turn on* all the lamps.
The office door is lock* in the evening.

In the first example, ‘turn on’ should be ‘turning on’ because it is after ‘are’.
In the second example, ‘lock’ should be ‘locked’ because it is after is.
The following chart outlines the use of verb forms after be.

VERB FORMS AFTER BE


BE + (1) present participle
(2) past participle

PRACTICE 2
Each of the following sentences contains a verb formed with be. Underline the verbs twice.
Then indicate if the sentences are correct (C) or incorrect (I).

1. She was study the textbooks all night long. ( I )


2. The pie was cut into six equal pieces. ( C )
3. Today the teacher is allow the students to leave class a few minutes early. ( )
4. The class is teach every other semester. ( )
5. Tom is bringing some drinks to the party. ( )
6. The sick child was taken to see a doctor. ( )
7. The children are swim in the backyard pool. ( )
8. The diamond jewelry is always keep in a safe place. ( )
9. The teacher is preparing a difficult exam for the students. ( )
10. Dinner is served from 6:00 to 8:00. ( )

3. BASE FORM VERBS AFTER MODALS

Whenever you see a modal such as will, would, shall, should, can, could, may, might, or
must, you should be sure that t he verb that follows it is in the base form.

You must telling* her the truth.


The child may comes* in now.

In the first example, ‘telling’ should be the base form ‘tell’ because it follows ‘must’.
In the second example, ‘comes’ should be the base form ‘come’ because it follows ‘may’.

The following chart outlines the use of verb forms after modals.

52
VERBS FORMS AFTER MODALS

MODAL + base form of the verb

PRACTICE 3

Each of the following sentences contains a verb formed with a modal. Underline the verbs
twice. Then indicate if the sentences are correct (C) or incorrect (I).

1. You should try to respond politely. ( C )


2. Everyone must leaves the room immediately. ( I )
3. I could sat on the beach for hours. ( )
4. She will asking you many difficult questions. ( ____ )
5. You can look at the book, but you cannot borrow it. ( )
6. He may lies to you because he is not very truthful. ( )
7. He knew that he would forgot the phone number. ( )
8. The weather man said that it might snowing tonight. ( )
9. Perhaps we could bought a new car this year. ( )
10. This course will satisfy the graduation requirement. ( )

PRACTICE 4

Each of the following sentences contains a verb formed with several parts. Underline the verbs
twice. Then indicate if the sentences are correct (C) or incorrect (I).

1. We have became good friends in the last year. ( )


2. Your name will be list in the new directory. ( )
3. The new movie is receive good reviews. ( ____ )
4. She must have feel sorry about her bad behavior. ( )
5. They have always given their family many presents. ( )
6. We may be taking a vacation next week. ( )
7. We could have taking a vacation last week. ( )
8. The package might had been deliver by an express mail service. ( )
9. I have not wrote very many letters to my friends. ( )
10. The car should not have be drive anymore yesterday. ( )

PRACTICE 5

Choose the letter of the underlined word or group of words that is not correct.

1. By the 1920s, many radio transmitters had been build.


A B C D

2. Fish farming has rose in the United States in recent years.


A B C D

3. In areas of volcanic activity, beach sand may contains dark minerals and little quartz.
A B C D

53
4. Cro-Magnon man was names after the caves in southwest France where the first remains
A B C
were discovered.
D

5. Lassie, the famous collie who made her first screen appearance in 1943, has always be
A B C
played by a male dog.
D

6. A blue bigwig lizard stakes out a territory and will defending females within it against court
A B C D
ing males.

7. President George Washington was inaugurates on the steps of the Federal Building in New
A B C D
York City.

8. By 1627, Plymouth had became a viable and growing community of fifty families, twenty-
A B C
two, goats, fifteen cows, and more than fifty pigs.
D
9. Tobacco was the crop on which the eminence of Williamsburg and the prosperity of Virginia
A B C
were base.
D
10. Because there may be scores of genes in each suspect DNA region, scientists must identi-
A B
fying and sequence the actual genes contributing to type I diabetes.
C D

PRACTICE 6

Choose the letter of the word or group of words that best completes the sentence.

1. the deepest valleys and canyons on the Earth.


A. In the Pacific Ocean with B. In the Pacific Ocean
C. The Pacific Ocean D. The Pacific Ocean has

2. In the United States, the participation of females in the labor force from 37
percent in 1965 to 51 percent in 1980.
A. it jumped B. jump
C. jumping D. jumped

Choose the letter of the underlined word or group of words that is not correct.

3. Water stored behind a dam can used to drive turbines.


A B C D

54
5. Our universe may continue to expand as it gets colder, empty, and deader.
A B C D

6. Every form of matter in the world are made up of atoms.


A B C D

7. The lens and cornea are supply with nutrients and oxygen by the aqueous fluid.
A B C D

8. Dodge City, laid out in 1872, owed both its prosperity and its famous to the buffalo in its
A B C
early years.
D

9. The amount of the two kinds of cholesterol in the blood have been shown to have an
A B C
effect on the risk of heart attack.
D

10. By the time Noah Webster reached his mid-twenties, he had already publish an elementary
A B C D
speller

B. READING

“WHERE” QUESTIONS
Sometimes the final question accompanying a reading passage (or one paragraph of a pas- sage)
on both the paper TOEFL test and the computer TOEFL test will ask you to determine where
in the passage a piece of information is found. This type of question is worded differently on
the paper TOEFL test and on the computer TOEFL test. On the paper test, there will be a
multiple choice question that asks where certain information is found. The answer choices will
list possible locations for that information.

Example:

The passage:

The words “capital” and “capitol” are confused in spelling and in meaning by a lot of people who
try to use them. Both their spellings and their meanings are quite closely related. A “capital is the
location of the center of government, while a “capitol” is the actual building where the
government officials meet. Thus, in the United States, for example, the Capitol building is lo-
cated in Washington D.C., which is the capital city of United States.

The question:
Where in the passage does the author define the word capital?
A. Lines 2-3 B. Lines 4-5 C. Line 6 D. Line 8

55
WHERE TO FIND THE ANSWER

The answer can be in any of the lines listed in the answers to the question.

HOW TOANSWER THE QUESTION

1. Choose a key word or idea in the question.


2. Skim the appropriate part(s) of the passage looking for the key word or idea.
3. Choose the answer that contains the key word or idea.

A geyser occurs when rainwater seeps into the ground and volcanic magma beneath
the surface heats it. The rainwater then turns into steam. The pressurized steam rises
to the surface and bursts out as a geyser. Yellowstone National Park has more geysers
than all of the rest of the world together.

5 The most famous of these geysers is Old Faithful, which erupts in a high arc of steam
about once an hour. There have not been any volcanic eruptions in the Yellowstone
area for 70,000 years. However, the existence of the geysers is proof that the area is
volcanically active.

1. Where in the passage does the author mention what heats the water in a geyser?
A. Lines 1-2 B. Line 4 C. Lines 5-6 D. Line 7

2. Where in the passage does the author state how long it has been since a volcano erupted at
Yellowstone?
A. Line 2 B. Line 4 C. Lines 5-6 D. Lines 7-8

56
CHAPTER 9
Short Dialogues

Short dialogues appear on both the paper TOEFL test and the computer TOEFL test. Though short dialogues are slightly
different in format on the two tests, they both test the same language skills. The paper and computer short dialogues are
similar in the following ways:
 the language skills tested
 the type of question used
 the number of people talking
The paper and computer short dialogues are different in the following ways:
 the possible number of lines of dialogue
 the use of context-setting visuals to accompany the dialogues
 the control of the timing between questions
 the presentation of the question

A. SHORT DIALOGUES ON THE PAPER TOEFL ®TEST

Short dialogues are found in Part A in the Listening Comprehension section of the paper TOEFL test. For each of the 30
short dialogues in this part of the test, you will hear a two-line dialogue between two speakers followed by a multiple-
choice question. After you listen to the dialogue and the question, you must choose the best answer to the question from
your test book. Look at an example of a short dialogue from the paper TOEFL test.

Example from the Paper TOEFL® Test.


On the recording, you hear: In your test book, you read:
man : This physics course couldn't be any harder. (A) She has something to say to the man.
Woman: I'll say! (B) She doesn't think the physics course is
Narrator: What does the woman mean? hard.
(C) She agrees with the man.
(D) She'd like to discuss the physics course.
In the dialogue, when the woman says “I’ll say”, she is showing that she agrees with what the man just said. Answer
(C) is therefore the best answer to this question.

B. PROCEDURES FORTHE SHORT DIALOGUES (Paper TOEFL® Test)


1. As you listen to each short dialogue, focus on the second line of the conversation. The answer
to the question is generally found in the second line.
2. Keep in mind that the correct answer is probably a restatement of a key word or idea in the
second line of the dialogue. Think of possible restatements.
3. Keep in mind that certain structures and expressions are tested regularly in the short dialogues.
Listen for these structures and expressions:
• structures (passives, negatives, wishes, conditions)
• functional expressions (agreement, uncertainty, suggestion, surprise)
• idiomatic expressions (two-part verbs, three-part verbs, idioms)
4. Keep in mind that these questions generally progress from easy to difficult. This means that
questions I through 5 will be the easiest and questions 26 through 30 will be the hardest.
5. Read the answers and choose the best answer to each question. Remember to answer each
question even if you are not sure of the correct response. Never leave any answers blank.
6. Even if you do not understand the complete dialogue, you can still find the correct answer.
• If you only understand a few words or ideas in the second line, choose the answer that contains
a restatement of those words or ideas.
• If you do not understand anything at all in the second line of the conversation, choose the
answer that sounds the most different from what you heard.
• Never choose an answer because it sounds like what you heard in the dialogue.
7. Be prepared for the next question. You have only 12 seconds be tween questions.

57
C. STRATEGIES

SKILL I: FOCUS ON THE LAST LINE

The short dialogues involve conversations between two people, each followed by a question. It is important to
understand that the answer to this type of question is most often (but not always!) found in the last line of the
conversation.

Example from the Paper and Computer TOEFL® Tests.


On the recording, you hear: In your test book or on the computer screen, you
read:
man : Billy really made a big mistake this time. (A) It was the first time he made a mistake.
woman : Yes, he forgot to turn in his research paper (B) He forgot to write his paper.
narrator : What does the woman say about Billy? (C) He turned in the paper in the wrong place.
(D) He didn't remember to submit his assignment.
The last line of this dialogue indicates that Billy forgot to turn in his research paper, and this means that he didn't
remember to submit it. The best answer is therefore answer (D).
The following chart outlines the most important strategy for the short dialogues.

STRATEGY #1: FOCUS ON THE LAST LINE


1. The last line of the dialogue probably contains the answer to the question.
2. Listen to the first line of the dialogue. If you understand it, that’s good. If you don’t understand it, don’t worry
because it probably does not contain the answer.
3. Be ready to focus on the last line of the dialogue because it probably contains the answer. Repeat the last line
in your mind as you read through the answers in the text.

EXERCISE 1: In this exercise, you should focus on the last line of the dialogue, read the question, and then
choose the best answer to that question. Remember that you can probably answer the question easily with only
the last line.

1. (man) Can you tell me if today's matinee is (A) She has strong ideas about movies.
a comedy, romance, or western? (B) She prefers comedies over westerns and
romances.
(woman) 1 have no idea (C) She doesn't like today's matinee.
(narrator) What does the woman mean? (D) She does not know.

2. (woman) Was anyone at home at Bath's house (A) Barb answered the bell.
when you went there to deliver (B) The house was probably empty.
the package? (C) The bell wasn't in the house.
(man) I rang the beg but no one answered. (D) The house doesn't have a bell.
(narrator) What does the man imply?

3. (woman) You just got back from the interview (A) It's unlikely that he'll go to the interview.
for the internship. How do you (B) He thinks he'll be recommended for
think it went? a high-level job.
(man) I think it’s highly unlikely (C) The interview was apparently quite
that I get the job. unsuccessful.
(narrator) What does the man suggest? (D) He had an excellent interview.

TOEFL EXERCISE 1: In this exercise, listen carefully to each short dialogue and question on the
recording, and then choose the best answer to the question. You should focus carefully on the last line.

58
NOW BEGIN THE RECORDING AT TOEFL EXERCISE 1.

1. (A) He is leaving now. (C) He will not be leaving soon.


(B) He has to go out of his way. (D) He will do it his own way.

2. (A) He locked the door. (C) He was able to open the door.
(B) He tried unsuccessfully to get into the house. (D) He left the house without locking the door.

3. (A) She doesn't like to listen to turkeys. (C) She especially likes the roast turkey.
(B) She thinks the dinner sounds special. (D) She'd prefer a different dinner.

4. (A) He'll be busy with her homework tonight. (C) He's sorry he can't ever help her.
(B) He can't help her tonight. (D) He'll help herwith her physics.

5. (A) Her eyes hurt. (C) The class was boring.


(B) She thought the lecture was great. (D) She didn't want to watch Professor Martin.

6. (A) Not all the bills have been paid. (C) What she said on the phone wasn’t credible.
(B) They don’t have enough credit to pay the bills. (D) He used a credit card to pay some of the bills.

7. (A) She’ll call back quickly. (C) She ‘ll give it back by 4.00.
(B) She ‘ll definitely be back by 4.00 (D) She’ll try to return fast.

8 . (A) She hasn’ t seen Tim. (C) Tim was around a short t ime a go.
(B) Tim was t her e only f or a moment. (D) Tim will r eturn i n a minute.

9 . (A) She doesn’t l ike the place he c hooses. (C) She’s glad the spot is res er ved
(B) She doesn’t want to get i nto the c ar. (D) They c an’ t park t he c ar t her e.

1 0. (A) There’s plenty to eat. (C) The f ood isn’t in t he r ef rige rator.
(B) The ref riger ator’s broken. (D) He isn’t sure if the re’ s e nough.

SK ILL 2: CHOOSE ANSWERS WITH SYNONYMS

Often the correct answer in a short dialogue is an answer that contains synonyms (words with similar meanings but
different sounds) for key words in the conversation.

Example from the Paper and Computer TOEFL® Tests.


On the recording, you hear: In your test book or on the computer screen, you
read:

woman : Why is Barbara feeling so happy? (A) She always liked her work in real estate.
man : She just started working in a real estate (B) She began a new job.
agency. (C) She just bought some real estate.
narrator : What does the man say about Barbara? (D) She bought a real estate agency.
In this dialogue, the key word started means began, and the key word working refers to job. The best answer to this
question is therefore answer (B).

The following chart outlines a very important strategy for short dialogues:
STRATEGY #2: CHOOSE ANSWERS WITH SYNONYMS

1. As you listen to the last line of the dialogue, focus on the key words of that line.
2. If you see any synonym for key words in a particular answer, then you have probably found the correct
answer.

59
EXERCISE 2: In this exercise, underline key words in the last line of each short dialogue. Then underline
synonyms for these key words in the answers, and choose the best answer to each question. Remember that the best
answer is probably the answer that contains synonyms for the key words in the last line of the dialogue.

1. (woman) Did you see the manager (A) He got a job as bookstore manager.
about the job in the bookstore? (B) The bookstore was not accepting applications.
(man) Yes, and I also had to fill out an (C) He saw a book about how to apply for jobs.
application. (D) It was necessary to complete a form.
(narrator) What does the man mean?

2. (man) We're planning to leave for the trip (A) If they leave at noon.
at about 2:00. (B) If it is possible to go by 12.00.
(woman) Couldn't we leave before noon? (C) Why they can’t leave at noon.
(narrator) What does the woman ask? (D) If they could leave the room.

3. (man) Was the concert well-received? (A) The performance went on for a long time.
(woman) The audience applauded for (B) There was applause throughout the
a long time after the performance. performance.
(narrator) What does the woman say about (C) The people clapped on and on after
the concert? the concert.
(D) The audience waited for a long time for
the concert to begin.

TOEFL EXERCISE 2: In this exercise, listen carefully to each short dialogue and question on the recording, and then
choose the best answer to the question. You should look for synonyms for key words in the last line.

NOW BEGIN THE RECORDING AT TOEFL EXERCISE 2.

1. (A) The final exam was harder than the others. (C) He thought the exam would be easier.
(B) There were two exams rather than one. (D) The exam was not very difficult.

2. (A) He's not feeling very well. (C) He’s feeling better today than yesterday.
(B) He's rather sick of working. (D) He’d really rather not answer the question.

3. (A) The company was founded about a year ago. (C) The family is well-established.
(B) It was just established that he could go into (D) The business only lasted a year.
business.

4. (A) He did not look at the right schedule. (C) The plane arrived on time.
(B) The plane landed in the right place. (D) He had to wait for the plane to land.

5. (A) She'd rather go running. (C) She’ll change clothes quickly and go swimming.
(B) She doesn't want to go into the pool. (D) She needs a sweat suit to go running.

6. (A) The firefighters saved the homes for last. (C) The homes on the hillside were burned.
(B) A firefighter saved the hillside last night. (D) The houses weren’t destroyed.

7. (A) there’s enough soup. (C) She thinks the soup’s too salty.
(B) The spaces are adequate. (D) The man should add more salt and pepper.

8. (A) He was lucky to receive a grant for (C) He is a scholar at collage with low fees.
his studies.
(B) He used his fortune to pay his fee. (D) He paid to get a scholarship.

9. (A) It profited from previous mistakes. (C) This was the last year that it would make a profit.
(B) It earned a lot of money. (D) It was not successful.
60
10. (A) Chucks bank account has too much (C) He thinks that Chuck is on his way home from
money in it. the bank
(B) He thinks Chucks has the wrong (D) there isn’t enough money in Chucks’ account.
kind of bank account.

SKILL 3: AVOID SIMILAR SOUNDS

Often the incorrect answers in the short dialogues are answers that contain words with similar sounds but very different
meanings from what you hear on the recording. You should definitely avoid these answers.

Example from the Paper and Computer TOEFL® Tests


On the recording, you hear: In your test book or on the computer screen, you
read:

man : Why couldn't Mark come with us? (A) He was in the department office.
woman : He was searching for a new apartment. (B) He was looking for a place to live.
narrator : What does the woman say about Mark? (C) He was working on his research project.
(D) He had an appointment at church.
The key words in the last line of the dialogue are searching and apartment. In answers (C) and (D), the words
research and church sound like search, so these answers are incorrect. In answers (A) and (D), the words
department and appointment sound like apartment, so these answers are incorrect. The best answer is therefore
answer (B).

The following chart outlines a very important strategy for the short dialogues:
STRATEGY #3: AVOID SIMILAR SOUNDS

1. Identify key words in the last line of the dialogue.


2. Identify words in the answers that contain similar sounds and do not choose these answers.

EXERCISE 3: In this exercise, underline key words in the last line of each short dialogue. Then underline words
with sounds similar to these key words in the answers, and choose the best answer to each question. Remember that
the best answer is probably the answer that does not contain words with sounds that are similar to the sounds of the
key words in the last line of the dialogue.

1. (woman) I heard that Sally just moved (A) Sally has no sense of responsibility.
into a new, big house near the beach. (B) Sally sent her friend to the house.
(man) But Sally doesn't have a cent! (C) Sally has no money.
(narrator) What does the man mean? (D) Sally is on the set with her.

2. (woman) Did they get the new car they wanted? (A) They locked the map in a car.
(man) No, they lacked the money. (B) They looked many times in the car.
(narrator) What does the man mean? (C) It cost a lot of money when the car leaked oil
(D) They didn't have enough money to buy
another car.

3. (man) Have you finished packing yet? (A) It’s important to pack the suitcases.
(woman) You should call the porter to get (B) They need help carrying their bags.
the suitcases. (C) The man should pack his suit in case he needs it.
(narrator) What does the woman mean? (D) The suitcases are quite portable.

TOEFL EXERCISE 3: In this exercise, listen carefully to each short dialogue and question on the recording, and then
choose the best answer to the question. You should be careful to avoid answers with similar sounds.

61
NOW BEGIN THE RECORDING AT TOEFL EXERCISE 3.

1. (A) She has to wait for some cash. (C) The lawn is too dry.
(B) The waiter is bringing a glass of water. (D) She needs to watch out for a crash.

2. (A) The sweater's the wrong size. (C) The sweater makes the man seem fat.
(B) The man's feet aren't sweating. (D) The sweet girl doesn't feel right.

3. (A) He has been regularly using a computer. (C) He regularly goes to communities around Boston.
(B) He communicates with a Boston company. (D) He has been traveling back and forth to Boston.

4. (A) He thought the lesson didn't matter. (C) He learned a massive number of details.
(B) He couldn't learn the lesson. (D) He didn't like most of the lesson.

5. (A) Some animals started the first fire. (C) In the first frost, animals die.
(B) Animals are killed by forest fires. (D) Frost can kill animals.

6. (A) Twenty pairs of shoes are on sale. (C) The shoes cost twenty dollars.
(B) The shoe salesclerk spent twenty (D) The shoes could be repaired for twenty
Dollars on pears. Dollars.

7. (A) Tom tended to dislike biology lab. (C) Tom went to biology lab.
(B) Attendance wasn’t necessary at biology lab. (D) There was a tendency to require biology lab.

8. (A) The meal will be served at noon. (C) He’s expecting the ice to melt before noon.
(B) The males should be driven there by noon (D) The letters ought to be delivered at 12.00

9. (A) The weather will probably get worse later. (C) The newspaper headlines
(B) The newspaper headlines described a bad (D) He had a new bad.
storm.

10. (A) If she could do the grocery shopping. (C) If she could help prepare the salad.
(B) If she prefers cooked vegetables or salad (D) If she minds shopping for vegetables.

TOEFL EXERCISE (Skills 1-3): In this exercise, listen carefully to each short dialogue and question on the
recording, and then choose the best answer to the question.

NOW BEGIN THE RECORDING AT TOEFL EXERCISE (SKILLS 1-3).

1. (A) He would like some iced coffee. (C) A drink seems like a good idea.
(B) He wants to stop drinking coffee. (D) He needs to drink something to stop his coughing.

2. (A) She would prefer a sunny day. (C) She would like a place that is not so loud.
(B)The park is too crowded. (D) She cannot walk because she's too old.

3. (A) He should open an account. (C) He should try to keep the cost cheap.
(B) He should take a ride on a ship. (D) He should try something monotonous to get to sleep.

4. (A) The department is not changing the requirements. (C) The changes are believable.
(B) He hasn't heard anything about the change. (D) What has happened is incredible to him.

5. (A) The wait has taken close to an hour. (C) Most of the people have been in line for hours.
(B) They were stranded in their car. (D) They made a line in the sand.

62
6. (A) The instructor is selecting several passages. (C) The stamp collector is conducting his business.
(B) The conductor is fair to the passengers. (D)The riders are paying for the train trip.

7. (A) The managers will take the train to the program. (C) The new management program is very
weak.
(B) A program to develop new managers will (D) The program will be maintained to the letter.
commence soon.

8. (A) The fire started to attack the building. (C) The fire probably began at the top of the building.
(B) The firefighter stared at the attacker. (D) The firefighter started to attack the fire.

9. (A) He assured the woman that he knew the truth. (C) He thought that the woman was aware of what
happened.
(B) He is sure that it isn't new. (D) He soon will know the truth.

10. (A) The art professor is not one of his fans. (C) The catches that he made were fantastic.
(B) His drawings were amazing. (D) His sketches showed a fantasy world.

S K IL L 4 : D R A W C O N C L U S I O N S A B O U T W H O , WHAT, WHERE

It is common in the short dialogues to ask you to draw some kind of conclusion. In this type of question, the answer is
not clearly stated; instead you must draw a conclusion based on clues given in the dialogue. One kind of conclusion that
is common in this part of the test is to ask you to determine who the speaker is, based on clues given in the dialogue.

Example from the Paper and Computer TOEFL® Tests


On the recording, you hear: In your test book or on the computer screen, you
read:

woman: Can you tell me what assignments I missed (A) A newspaper editor
when I was absent from your class? (B) A police officer
man : You missed one homework assignment and a (C) A teacher
quiz. (D) A student
narrator : Who is the man?

The clues class, homework, and quiz in the dialogue tell you that the man is probably a teacher. Answer (C) is therefore the
correct answer.
Another type of conclusion that is common in the short dialogues is to determine what will probably happen next, based
on clues given in the dialogue.

Example from the Paper and Computer TOEFL® Tests


On the recording, you hear: In your test book or on the computer screen, you
read:

woman : Are you going to read those books here in the (A) Sit down in the library
library? (B) Look for some more books
man : I think I'd rather check them out now and take (C) Return the books to the shelves
them home. (D) Go to the circulation desk
narrator : What will the man probably do next?
The man mentions books and says that he would like to check them out now. Since the circulation desk is where you go to
check books out from a library, the man will probably go to the circulation desk next. The correct answer is therefore
answer (D). A final type of conclusion that is common in the short dialogues is to determine where the conversation
probably takes place, based on clues given in the conversation.

63
Example from the Paper and Computer TOEFL® Tests
On the recording, you hear: In your test book or on the computer screen, you
read:

woman : Are you going into the water, or are you (A) At a beauty salon
just going to lay there on the sand? (B) At the beach
man : I think I need to put on some sun and lotion. (C) In a s a nd b ox
narrator : Where does this conversation probably take (D) At an outdoor restaurant
place?
The clues water, sand, and suntan lotion in the dialogue tell you that this dialogue probably takes place at the
beach. Answer (B) is therefore the correct answer.

The following chart outlines the key point that you should remember about this type of question:
CONCLUSIONS ABOUT WHO, WHAT, WHERE

It is common for you to be asked to draw one of the following conclusions in the short dialogues
1. WHO is probably talking
2. WHAT will s/he probably do next?
3. WHERE does the dialogue probably take place?

EXERCISE 4: In this exercise, read each short dialogue and question, underline the clues that help you answer the
question, and then choose the best answer. You will have to draw conclusions about who, what, and where.

1. (man) I'd like to deposit this check in (A) A store clerk


my account, please. (B) A bank teller
(woman) Would you like any cash back? (C) An accountant
(narrator) Who is the woman? (D) A waitress

2 . (woman) Have you deposited your (A) Earn his paycheck


paycheck yet? (B) Write a check for a deposit on an apartment
(man) No, but that's next on my list of (C) Go to a bank
errands. (D) Make a list of errands to run
(narrator) What will the man probably do
next?

3 . (man) Did you get the bread, eggs, and milk? (A) In a restaurant
(woman) Now we need to stand in line (B) At a b a ke r y
at the checkout counter. (C) O n a f a r m
(narrator) Where does this conversation (D) In a m a r ke t
probably take place?

TOEFL EXERCISE 4: In this exercise, listen carefully to each short dialogue and question on the recording and then
choose the best answer to the question. You will have to draw conclusions about who, what, and where.

NOW BEGIN THE RECORDING AT TOEFL EXERCISE 4 .

1. (A) In a photography studio (C) In an office


(B) In a biology laboratory (D) In the library

2. (A) He's a pilot. (C) He’s a member of the ground crew


(B) He's a flight attendant. (D) He works clearing land.

64
3. (A) Wash the dishes immediately (C) Wash the dishes for as long as possible
(B) Use as many dishes as possible (D) Wait until later to clean up

4. (A) In a bank (C) At a service station


(B) In a restaurant (D) In a beauty salon

5. (A) A salesclerk in a shoe store (C) A party caterer


(B) A shoe repair person (D) A sales clerk in a fixtures department

6. (A) On a playground (C) At a zoo


(B) In a parking lot (D) In a photo studio

7. (A) Respond to the mail (C) Create a pending file


(B) Put the letters in a file (D) File the answers she received to the
letters.

8. (A) In an airplane (C) In a theatre


(B) In a police car (D) At a fireworks exhibit

9. (A) Take care of Bob (C) Let Bob know that they accept his invitation
(B) Invite Bob to d inner (D) Respond to the woman’s question

10.(A) A pharmacist (C) A teacher


(B) A dentist (D)A business manager

SKILL 5: LISTEN FOR WHO AND WHAT IN PASSIVES

It is sometimes difficult to understand who or what is doing the action in a passive sentence. This problem is often
tested in the short dialogues.

Example from the Paper and Computer TOEFL® Tests


On the recording, you hear: In your test book or on the computer screen, you
read:
man : Did Sally go to the bank this morning? (A) Sally wrote several checks.
woman : Yes, she did. She got a new checking account. (B) Sally wanted to check up on the bank.
narrator : What does the woman imply? (C) A new checking account was opened.
(D) Sally checked on the balance in her
account.
In this dialogue, the woman uses the active statement She got a new checking account, which means that Sally opened
a checking account. The correct answer uses the passive structure that a new checking account was opened to express
the same idea. Therefore, the best answer to the question above is answer (C).

You should note the following about passive sentences in the short dialogues:
PASSIVE STATEMENTS

1. If the dialogue contains a passive statement, the answer to the question is often an active statement.
2. If the dialogue contains an active statement, the answer to the question is often a passive statement.

NOTE: Check carefully who or what is doing the action in these questions.

EXERCISE 5: In this exercise each of the correct answers is either a passive restatement of an active sentence or an
active restatement of a passive sentence. Read each short dialogue and underline the key active or passive statement.
Then read the question and choose the best answer to the question. Be careful about who and what with these
passives.

65
1. (woman) Alice needs to pay her tuition today. (A) Alice's education has paid off.
(man) But her tuition has already been paid. (B) Alice's tuition needs to be paid.
(narrator) What does the man imply? (C) Alice has already paid her fees.
(D) Alice has already received the money.

2. (man) Have you been taking good care of (A) She drank some water on the lawn this morning.
the lawn?
(woman) I watered it only this morning. (B) She waited for him on the lawn this morning.
(narrator) What does the woman mean? (C) The lawn has already been watered today.
(D) She wanted a new lawn this morning.

3. (man) Did you hear the news about the child (A) Someone located the girl.
who was lost in the park? (B) She heard about the new park from the child.
(woman) Yes, and I heard that she was just found! (C) The child found her lost pet.
(narrator) What does the woman mean? (D) The child was the last one in the park.

TOEFL EXERCISE 5: In this exercise, listen carefully to each short dialogue and question on the recording, and
then choose the best answer to the question. You should be particularly careful of passives.

NOW BEGIN THE RECORDING AT TOEFL EXERCISE 5.

1. (A) If the restaurant is on the corner


(B) If the man would like to go to the restaurant
(C) If the vegetables are fresh
(D) If vegetarian food can be obtained

2. (A) He admitted that he wanted to go to law school in the fall.


(B) The law school accepted him as a student.
(C) The law professor admitted that he would be a student in the fall semester.
(D) He would be admitted to law school after the fall semester.

3. (A) Mark's plants were cared for in his absence.


(B) Mark's plan was to be out of town.
(C) Mark was careful about his plans for the out of town trip.
(D) She was careful while Mark was gone.

4. (A) The lights in the trees were destroyed in the storm.


(B) The storm damaged the trees.
(C) The falling trees destroyed a store.
(D) In the light the destruction of the storm could be seen.

5. (A) She was broke from skiing.


(B) She went skiing in spite of her accident.
(C) Her leg was hurt on a skiing trip.
(D) Her skis were broken in the mountains.

6. (A) The road the horses took was long and hard.
(B) It was hard to find the hidden houses.
(C) The riders worked the horses too much.
( D) It was hard for people to ride the horses for long.

7. (A) He didn’t want the coffee that the women ordered.


(B) He wasn’t sure if the women wanted coffee.
(C) He assumed the women had ordered coffee.
(D) He was unaware that coffee had already been ordered

66
8. (A) The car was in the left parking lot at the airport.
(B) The friends parked their car at the airport.
(C) The airport couldn’t hold a lot of cars.
(D) There were a lot of cars to the left of the parking lot.

9. (A) The student pointed at Mac.


(B) Mac was present when the other students made the appointment.
(C) The class representative suggested Mac to the other students.
(D) Mac was chosen by his classmates to represent them.

10. (A) After the earthquake, the insurance company came out to inspect the damage.
(B) The insurance company insisted that the building be repaired to meet earthquake safety
standards.
(C) The inhabitants paid their premium after the earthquake.
(D) The insurance company paid for the earthquake damage.

SKILL 6: LISTEN FOR WHO AND WHAT WITH MULTIPLE NOUNS

When there is more than one noun in a sentence in the short dialogues, it is common for the answers to confuse which
noun does what.

On the recording, you hear: In your test book or on the computer screen, you
read:

man : Do you know who is in the band now? (A) Robert became a new member of the
woman : I heard that Mara replaced Robert in the band.
band. (B) Robert took Mara's place in the band.
narrator : What does the woman say about the band? (C) Mara didn't have a place in the band.
(D) Mara took Robert's place in the band.

In the woman's response to the man's question, she talks about two people (Mara and Robert), and these two people are
confused in the answers. Because Mara replaced Robert, this means that Mara took Robert's place in the band. The best
answer is therefore answer (D).
The following chart outlines the key point that you should remember about questions with multiple nouns:

WHO AND WHAT WITH MULTIPLE MOUNS

When there are multiple nouns in a sentence, it is common for the answers to confuse which noun does what.

EXERCISE 6: In this exercise, underline the confusing nouns in each short dialogue. Then, read-the question
and choose the best answer to that question. Remember to think very carefully about who is doing what.

1. (man) Why is Bill not at work this week? (A) The doctor decided to take some time off
(woman) His doctor made him take a week off. from work.
(narrator) What does the woman mean? (B) The doctor told Bill he wasn't too weak to
work.
(C) Bill was mad when the doctor took some
time off.
(D) Bill took a vacation on his doctor's orders.

2 . (man) Why is Paul going back home this (A) Paul is getting married this summer.
summer? (B) Paul's sister is returning from Vermont
(woman) He's returning to Vermont for to get married.
his sister's wedding. (C) Paul will be there when his sister gets married
67
(narrator) What does the woman mean? this summer.
(D) Paul's sister is coming to his wedding in
Vermont.

3. (man) Did you hear that John's uncle died? (A) John received an inheritance when his uncle
(woman) Yes, and John was named beneficiary died.
in his uncle's will. (B) It's a benefit that John's name is the same as
(narrator) What does the woman mean? his uncle's.
(C) John knows that his uncle will come to the
benefit.
(D) John's uncle gave him a beneficial name.

TOEFL EXERCISE 6: In this exercise, listen carefully to each short dialogue and question on the recording, and
then choose the best answer to the question. You should be particularly careful of who is doing what.

NOW BEGIN THE RECORDING AT TOEFL EXERCISE 6.

1. (A) The passenger waited at the corner.


(B) The passenger looked for a taxi at the corner.
(C) The cab driver waited for the passenger.
(D) The p a s s enger co r ner ed t he waiting taxi driver.

2. (A) It was hard for her to hear Jane last night.


(B) Jane gave a harp recital last nig ht.
(C) Jane was playing hard while she was hurt.
(D) She played the harp last night for Jane.

3. (A) The baby sister went to bed quite early.


(B) The children were forced to go to bed early.
(C) The babysitter made the bed after the children got up.
(D) The babysitter did not stay up late.

4. (A) The man taught his son about football.


(B) The boy is receiving the ball from his dad.
(C) The ball is being tossed into the air by the boy.
(D) The man is playing with the ball in the sun.

5. (A) The students were told to go listen to the speaker.


(B) The professor attended that evening's lecture.
(C) The students were given directions to the lecture.
(D) The professor was directed to the lecture hall.

6. (A) The manager went to the supply room.


(B) The clerk set supplies on the floor.
(C) The clerk went to the supply room at manager’s request
(D) The clerk backed into the manager in the supply room

7. (A) The librarian was quite reserved with the students for two days.
(B) Within two days the librarian had the books for the students.
(C) The librarian reserved the books for the students.
(D) The students put the books on hold for two days.

8. (A) The chairman decided that Tony would serve on the board for another year.
(B) The chairman elected the board.
(C) The board decided Tony could be chairman after one year.
(D) Tony becomes chairman for one more year.
68
9. (A) The judge defended the murdered.
(B) The judge tried to protect the defendant from the murdered.
(C) The judge said that the defendant was a criminal.
(D) The defense couldn’t make judgments about the criminal

10. (A) The woman should announce the names of the committee of members.
(B) He is thankful to be appointed to the committee.
(C) He is sure about the time of the appointment with the committee.
(D) The woman will serve on the committee.

TOEFL EXERCISE (Skills 4-6): In this exercise, listen carefully to each short dialogue and question on the
recording, and then choose the best answer to the question.

NOW BEGIN THE RECORDING AT TOEFL EXERCISE (SKILLS 4-6).

1. (A) In a department store (C) At the post office


(B) In a stationery store (D) At the airport

2. (A) The teacher gave the students a hand. (C) The students got the papers from the office.
(B) The term papers were turned in. (D) The teacher handed the papers to the students.

3. (A) The attendant checked the oil in Mark's car. (C) Mark checked with the service station attendant.
(B) Mark checked to see if he had enough oil (D) Mark wrote a check to pay for the oil.
in his car.

4. (A) A delivery man (C) A clerk in a fast-food restaurant


(B) A f amo us chef ( D ) An airline steward

5. (A) They need new print for the additional copies. (C) Printers are needed for the additional copies
(B) They can make extra copies if necessary. (D) A dditional copies are needed immediately.

6. (A) The professor bought two books. (C) The students sold two books to the professor.
(B) The students had to purchase two books. (D) The students were required to read two books by the
professor.

7. (A) The doctor returned to the office. (C) The doctor will not return until next week.
(B) Jim asked the doctor to come to the office. (D) Jim was told to come back.

8. (A) Go to work in the lab (C) Have the samples delivered


(B) Sample the work from the lab (D) Send a note to the lab

9. (A) Mary became the new class president. (C) In place of Mary, Sue became senior class president.
(B) Sue took her place as class president. (D) The senior class president replaced Sue and Mary.

10.(A) The panel was analyzed on the television (C) The program featured a psychoanalyst.
program.
(B) A committee evaluated recent political events. (D) The panel discussed the television program.

TOEFL REVIEW EXERCISE (Skills 1-6): In this exercise, listen carefully to each short dialogue and question on the
recording, and then choose the best answer to the question.

NOW BEGIN THE RECORDING AT TOEFL REVIEW EXERCISE (SKILLS 1-6).


1. (A) He seemed to be rather hungry. (C) He was trying to hang the posters.
(B) She was quite angry at him. (D) She believes he was mad.

69
2. (A) The parents are going to stay up late. (C) Lately, the parents have not been so loud.
(B) The parents have given Hannah her allowance. (D) Hannah does not have to go to bed early.

3. (A) At a department store (C) At a collection agency


(B) At a service station (D) In a delivery room

4. (A) She just broke some eggs. (C) She is serious about the boat.
(B) They need to eat fast. (D) He has a choice to make.

5. (A) It was urgent that Ellen do her best. (C) He was encouraged by Ellen to try harder.
(B) He really urged Ellen to do more. (D) Ellen told him that she was trying to do better.

6. (A) The car stalled on the road. (C) Rob sold his car.
(B) Someone took the car. (D) Rob heard someone steal his car.

7. (A) Buying the bigger container (C) Taking a carton that is smaller
(B) Putting the milk in the cart (D) Getting the milk tomorrow instead

8. (A) The receptionist welcomed the business (C) The businesspeople were rather greedy.
people.
(B) The man created a shipping and receiving (D) The businesspeople greeted the receptionist.
business.

9. (A) The police officer was stationed near the tourist (C) The tourist became mad at the police station
(B) The tourist was forced to accompany the police officer. (D) The tourist stated that the police officer never
came.

10.(A) He hasn't seen her ideas. (C) He doesn't like the idea.
(B) It was a terrible deal. (D) It sounds magnificent to him.

SKILL 7: LISTEN FOR NEGATIVE EXPRESSIONS

Negative expressions are very common in the short dialogues, and the most common kind of correct response to a
negative statement is a positive statement containing a word with an opposite meaning.

Example from the Paper and Computer TOEFL® Tests


On the recording, you hear: In your test book or on the computer screen, you
read:

man : How did they get to their grandmother's (A) They drove rather quickly.
house in Maine in only five hours? (B) They couldn't have driven more slowly.
woman : They didn't drive slowly on the trip to (C) They wanted to travel slowly to Maine.
Maine. (D) They didn't drive to Maine.
narrator : What does the woman say about the trip?
The correct answer is answer (A). If they didn't drive slowly to Maine, this means that they drove rather quickly.
Notice that the correct answer uses quickly, the opposite of slowly. The answers that use slowly are not correct.

The following chart outlines the types of negative expressions that you should be careful of:

TYPES OF NEGATIVE EXPRESSION

Expression Example Correct Answer


Regular negative: not or n’t Tom is not sad about the results Not sad = Happy
70
Other negatives, nobody, none, never Nobody arrived on time. Nobody…on time= late
Sal never works hard Never works hard= lazy
Negatives prefixes: un-, in-, dis- The patient was insane Insane= not sane= crazy

EXERCISE 7: In this exercise, underline the negative in the last line of each short dialogue. Then read the
question and choose the best answer to that question. Remember that the best answer is one that uses an opposite
meaning.

1. (man) I can't seem to get the door unlocked. (A) The key in the drawer is on the right.
(woman) That isn't the right key for the door. (B) The man should write the message on the door.
(narrator) What does the woman mean? (C) The man has the wrong key.
(D) The right key isn't in the drawer.

2. (man) Were you pleased with last week's (A) The convention was disorganized.
convention? (B) She didn't plan to attend the convention.
(woman) Nothing went as planned. (C) She planned the convention last week.
(narrator) What does the woman mean? (D) She wasn't pleased with the last week of the
convention.

3. (woman) Are you planning to go to college (A) He definitely wants to go to college.


next year? (B) He is certain about his plans.
(man) I'm really unsure about the idea. (C) He’s hesitant about attending college.
(narrator) What does the man mean? (D) His idea is to go to college.

TOEFL EXERCISE 7: In this exercise, listen carefully to each short dialogue and question on the recording, and then
choose the best answer to the question. You should be particularly careful of negative expressions.

NOW BEGIN THE RECORDING AT TOEFL EXERCISE 7.


1. (A) She is very busy. (C) It is not necessary to take out the trash.
(B) She has lots of free time. (D) She will do it if she has time.

2. (A) The interview is very important. (C) What’s he's wearing to the interview is important.
(B) He is worried about the interview. (D) He is not concerned about the interview.

3. (A) He has almost all the notes. (C) He went to all the lectures but one.
(B) His attendance was perfect. (D) He missed more than one psychology class.

4. (A) They passed the library at 6:00. (C) The library closes at 6:00.
(B) The library opens at 6:00 in the summer. (D) You can't check out more than six books in the
summer.

5. (A) Water the plants once a day. (C) Water the plants often while the man is gone.
(B) Give the plants no more water. (D) Give the plants a limited amount of water.

6. (A) The service satisfied her. (C) She thought the service was bad.
(B) The food was worse than the service. (D) Neither the food nor the service was satisfying.

7. (A) He told his kids to leave. (C) He was joking when he told the woman to leave
(B) He seriously wanted the woman to go. (D) He left with the woman.

8. (A) The project will take all their effort. (C) It’s impossible to finish.
(B) They have no other work to do. (D) They aren't even close to finishing the project.

9. (A) She doesn't mind an hour more. (C) It’s better to stay than go.
(B) She'd rather stay more than an hour. (D) She prefers to leave.

71
10. (A) The service at the hotel wasn't too good. (C) The service at the hotel could have been improved.
(B) This hotel gave excellent service. (D) This hotel's service was the same as the service
at other hotels

SKILL 8: LISTEN FOR DOUBLE NEGATIVE EXPRESSIONS

It is possible for two negative ideas to appear in one sentence, and the result can be quite confusing.

Example from the Paper and Computer TOEFL® Tests


On the recording, you hear: In your test book or on the computer screen, you
read:

man : I can't believe the news that I heard (A) There's no possibility that the concert will take
about the concert. place.
woman : Well, it isn't impossible for the concert to (B) The concert will definitely not take place.
take place. (C) The concert might take place.
narrator : What does the woman say about the (D) The concert can't take place.
concert?
The correct answer to this question is answer (C). If it isn't impossible for the concert to take place, then it is possible,
and the modal might indicate possibility.

The following chart outlines the situations where double negatives can occur:
DOUBLE NEGATIVES

Situation Example Meaning


Negative word (e.g., not, no, none) He didn’t like the unclean office. Did not like unclean office=
and a negative prefix (e.g., in-, un-, liked clean office.
dis-).
Two negative verbs. It isn’t snowing, so they aren’t Implies that they would go if it
going to the mountains. were snowing.
Neither or not… either. Sue didn’t like the movie, and Both did not like the movie.
neither did Mark.

EXERCISE 8: In this exercise, underline the two negatives in the last line of each short dialogue. Then read the
question and choose the best answer to that question. Remember that two negatives can make the sentence positive.

1. (man) Paula, you worked so hard setting (A) She hopes everyone will be pleased.
up the field trip. (B) She knows no one is happy with what she has done.
(woman) I hope no one's unhappy with (C) She's arranged to take a trip because she's unhappy.
the arrangements. (D) Everyone's happy with the condition of the field.
(narrator) What does Paula mean?

2. (woman) How was your history exam? (A) He studied a lot and passed.
(man) I didn't study enough, so I didn't (B) He failed in spite of his effort.
do well. (C) He got a good grade even though he didn't study.
(narrator) What does the man mean? (D) His grade was poor because of inadequate
preparation.

3. (man) Were your friends able to get tickets (A) Although Mark couldn't get both tickets, Paul did.
for the concert? (B) Both were unable to obtain tickets.
(woman) Mark couldn't get tickets for the (C) Neither Mark nor Paul wanted to go to the concert.
concert, and neither could Paul. (D) Mark tried to get tickets, but Paul didn't.
(narrator) What does the woman mean?

72
TOEFL EXERCISE 8: In this exercise, listen carefully to each short dialogue and question on the recording, and
then choose the best answer to the question. You should be particularly careful of double negatives.

NOW BEGIN THE RECORDING AT TOEFL EXERCISE 8.

1. (A) He'll definitely be elected. (C) She has high hopes for his chances.
(B) The election is now complete. (D) It may happen.

2. (A) Both parts of his game were bad. (C) Some parts of his game were better than others.
(B) He served better than he volleyed. (D) He played rather well.

3. (A) It is a surprise that he was prepared. (C) He prepared a really big surprise.
(B) He was not ready, as usual. (D) His strong preparation came as no surprise.

4. (A) She felt good enough to go out. (C) She felt like dancing, so she went out with
everyone.
(B) She went out to get some medicine. (D) She stayed home because she was sick.

5. (A) She has problems that others aren't aware (C) She knows she's been a problem of.
(B) Others aren't aware of her problems. (D) She doesn't have a care in the world.

6. (A) Steve wanted to finish his paper, and so (C) Steve and Paul were busy doing their term papers.
did Paul.
(B) Both Steve's and Paul's papers were (D) When Steve wasn't able to finish his paper, Paul
incomplete. couldn't help.

7. (A) It wasn't George's responsibility to pay (C) George acted carelessly by not taking care of the
the bill. bill.
(B) Bill was irresponsible about paying George's (D) George took responsibility for the unpaid bill.
rent.

8. (A) It's fortunate that he was accepted. (C) Fortunately, the university didn't admit him.
(B) It's good that he wasn't admitted. (D) It’s too bad he was rejected.

9. (A) The first essay was better than the (C) The second draft of the essay was much
second. better than the first.
(B) The first and second drafts couldn't (D) Both versions were poorly written.
be better.

10. (A) Roger has been bothered. (C) The problems have had little effect on Roger.
(B) Roger wasn't the least bit disturbed (D) Roger hasn't been disturbed.

73
CHAPTER 10
Long Talks

SK ILL 9: LISTEN FOR "ALMOST NEGATIVE" EXPRESSIONS


Certain expressions in English have "almost negative" meanings. These expressions are common in the short
dialogues.

Example from the Paper and Computer TOEFL ® Tests


On the recording, you hear: In your test book or on the computer screen, you
read:

woman : Were you able to pay the electric bill? (A) He had plenty of money for the bill.
man : I had barely enough money. (B) He did not have enough money for the bill.
narrator : What does the man imply? (C) He paid the bill but has no money left.
(D) He was unable to pay the bill.
In the man's statement, the word enough indicates that there was enough, so he paid the bill. However, it was barely
enough, so he almost did not have enough and certainly has no money left. The correct answer is therefore answer (C).

The following chart outlines common "almost negative" expressions:


COMMON ALMOST NEGATIVE EXPRESSION

Meaning Expression Example


almost none Hardly, barely, scarcely, only There is hardly any food in the refrigerator
almost never Rarely, seldom He rarely drives to work

EXERCISE 9: In this exercise, underline the "almost negative" expression in the last line of each short dialogue.
Then read the question and choose the best answer. Remember that the best answer is one that means that it is true
but it is almost not true.

1. (man) I hear that Mona's been offered (A) Mona hasn't worked hard.
the manager's job. (B) Mona's experience has been hard.
(woman) But she has hardly any work (C) Mona's job as manager is hard.
experience! (D) Mona hasn't worked for very long.
(narrator) What does the woman say
about Mona?

2. (woman) How much time did Sam spend (A) Sam usually spends this much time on his
on his paper for economics class? schoolwork.
(man) Sam has seldom taken so much (B) Sam has rarely worked so hard.
time on a research paper. (C) Sam took too much time on this paper.
(narrator) What does the man mean? (D) Sam should've worked harder on this paper.

3. (woman) Does Steve usually park his car (A) He parks his car there once in a while.
there? (B) He's parked his car there a lot.
(man) Only once has he parked his car (C) He only leaves his car there for short periods
in that lot. of time.
(narrator) What does the man mean? (D) He left his car there on just one occasion.

TOEFL EXERCISE 9: In this exercise, listen carefully to each short dialogue and question on the recording, and
then choose the best answer to the question. You should be particularly careful of "almost negative" expressions.

74
NOW BEGIN THE RECORDING AT TOEFL EXERCISE 9.

1. (A) There's little rain in July. (C) It rains hard in July.


(B) In July it never rains. (D) When it rains in July, it rains hard.

2. (A) The university accepted three students. (C) John was not accepted.
(B) None of the students is going to the (D) Two were not admitted.
university.

3. (A) Although he did pass, Mark's exam grade (C) The highest grade on the history exam went to Mark.
wasn't too good.
(B) Mark failed his history exam. (D) Professor Franks didn't pass Mark on the history
exam.

4. (A) He often has long waits in Dr. Roberts's (C) Dr. Roberts is generally punctual.
office.
(B) He must wait patiently for Robert. (D) He doesn't mind waiting for Dr. Roberts.

5. (A) Betty often takes vacations in winter. (C) Occasionally Betty works one week during
vacation.
(B) Betty prefers to take vacations in winter. (D) A winter vacation is unusual for Betty.

6. (A) He rarely spends time on his courses. (C) He never studies.


(B) He's an excellent student. (D) His books are always open.

7. (A) He finished the exam in plenty of time. (C) He used every possible minute to finish.
(B) He was scared he wouldn't finish. (D) He was unable to complete the exam.

8. (A) This was a very long staff meeting. (C) The meeting lasted only until one o'clock.
(B) This was the only staff meeting in a long time. (D) The one staff meeting should've lasted longer.

9. (A) Meat tastes delicious to him when it's (C) This meat is the best he's tasted in a long time.
cooked rare.
(B) He isn't sure if the meal is delicious. (D) He’d like to eat some meat from this delicatessen.

10. (A) He broke his arm trying to move it. (C) He only tries to move the broken arm.
(B) He only hurt the broken arm. (D) There’s no pain if he rests quietly.

SKILL 10: LISTEN FOR NEGATIVES WITH COMPARATIVES

Negatives can be used with comparatives in the short dialogues of the TOEFL test. A sentence with a negative and
a comparative has a superlative, or very strong, meaning.

Example from the Paper and Computer TOEFL® Tests.


On the recording, you hear: In your test book or on the computer screen, you
read:

woman : What do you think of the new student in (A) She is not very smart.
math class? (B) He is smarter than she is.
man : No one is more intelligent than she is. (C) Other students are smarter than she is.
narrator : What does the man say about the new (D) She is the smartest student in the class.
student?
The man responds to the woman's question with the negative No and the comparative more intelligent, and this combination
has a superlative meaning, the smartest. The best answer is therefore answer (D).

75
The following chart outlines comparisons that you should be careful of when they are used with negatives:
COMPARATIVES WITH NEGATIVES

Comparative Example Meaning


More No one is more beautiful than she is She is the most beautiful.
-er He couldn’t be happier. He is extremely happy.

EXERCISE 10: In this exercise, underline the negative and the comparative in the second line of each short
dialogue. Then read the question and choose the best answer to that question. Remember that the best answer is one
that expresses a superlative, or very strong, idea.

1. (woman) Have you gotten over your cold yet? (A) He's feeling terrific.
(man) I couldn't be feeling any better today. (B) He felt a lot worse today.
(narrator) What does the man mean? (C) He's not feeling too well today.
(D) He's a bit better today.

2. (woman) What did you think of Mike when you (A) Mike was extremely friendly when he met
first met him? him.
(man) He couldn't have been more (B) Mike could have met him sooner.
unfriendly. (C) Mike didn't seem to like him at all.
(narrator) What does the man mean? (D) When he met Mike, he didn't have a friend.

3. (man) Did you see Theresa's grade on the (A) Theresa could've gotten a higher grade.
math exam? It was unbelievable! (B) Anyone could get a good grade.
(woman) No one else could have done better (C) Theresa got the highest grade.
(narrator) Mat does the woman mean? (D) A high grade is impossible for anyone.

TOEFL EXERCISE 10: In this exercise, listen carefully to each short dialogue and question on the recording, and
then choose the best answer to the question. You should be particularly careful of comparatives with negatives.

NOW BEGIN THE RECORDING AT TOEFL EXERCISE 10.

1. (A) She's not very happy. (C) She could be somewhat happier.
(B) She didn't do very well on the exam. (D) She's delighted with the results.

2. (A) Paula is always lazy. (C) Paula made a strong effort.


(B) Paula didn't work very hard this semester. (D) Paula could have worked harder.

3. (A) The prices were great! (C) She didn't buy much because of the prices.
(B) The prices were too high. (D) The prices could have been lower.

4. (A) She is not very smart. (C) He doesn't know her very well.
(B) She always tells him everything. (D) She's extremely intelligent.

5. (A) The patient absolutely didn't need the (C) The surgeon felt that the operation was
surgery. necessary.
(B) The necessity for the surgery was (D) It was essential that the surgery be performed
unquestionable. immediately.

6. (A) They were not very lucky. (C) The accident was unfortunate.
(B) No one was hurt. (D) She wanted to have better luck.

7. (A) Nothing was very difficult. (C) The exam couldn't have been easier.
(B) The exam wasn't at all easy. (D) The exam had nothing difficult on it.

76
8. (A) She wants that job very much. (C) Everybody else wants that job as much as she
does.
(B) No one is going to get the job. (D) She is not sure about taking the job.

9. (A) She was second in the race. (C) She won the race.
(B) She was almost the slowest person in the (D) She was not faster than anyone else.
race.

10. (A) This math project was extremely complex. (C) They seldom complete their math projects.
(B) This math project was less complicated than (D) Complicated math projects are often assigned.
the last.

TOEFL EXERCISE (Skills 7-10): In this exercise, listen carefully to each short dialogue and question on the
recording, and then choose the best answer to the question.

NOW BEGIN THE RECORDING AT TOEFL EXERCISE (SKILLS 7-10).

1. (A) She can try a little harder. (C) She's doing the best that she can.
(B) There is a lot more that she can do. (D) It is impossible for her to do anything.

2. (A) She's always been late for the bus. (C) The bus only left on time once.
(B) The bus has always been late. (D) Only on this trip has the bus been on time.

3. (A) There wasn't enough soup to go around. (C) Everyone got one serving of soup, but there
wasn't enough for seconds.
(B) We had so much soup that we couldn't (D) Everyone around the table had a lot of soup.
finish it.

4. (A) She does want to see the movie. (C) She doesn’t want to go there anymore.
(B) It’s extremely important to her to go. (D) She really couldn’t move there.

5. (A) She handed the paper in on time. (C) The paper was a complete mess, so she didn't
turn it in.
(B) She was able to complete the paper, but she (D) The paper was unfinished.
didn't turn it in.

6. (A) Neither Tim nor Sylvia is taking care of Art. (C) Sylvia doesn't care for anything Tim does.
(B) Sylvia likes modern art even less than Tim (D) Sylvia and Tim agree in their opinion of
does. modern art.

7. (A) They always work hard in the afternoon. (C) After noon they never work.
(B) They don't do much after lunch. (D) It's never hard for them to work in the
afternoon.

8. (A) It's hard for him to work when it gets warm. (C) The air-conditioner only works when it isn't
needed.
(B) Whenever it gets warm, he turns on the air- (D) He likes to use the air-conditioner when it is
conditioner. warm.

9. (A) He did really poorly. (C) The results could not have been better.
(B) He's felt worse before. (D) He's not too unhappy with the results.

10. (A) With so many members present, the (C) The issue shouldn't have been decided by all
committee couldn't reach a decision. the committee members.
(B) The committee should've waited until more (D) The issue wasn't decided because so many
members were present. Members were absent.

77
SKILL 11: LISTEN FOR EXPRESSIONS OF AGREEMENT

Expressions of agreement are common in the short dialogues, so you should become familiar with them. The
following example shows agreement with a positive statement.

Example from the Paper and Computer TOEFL® Tests


On the recording, you hear: In your test book or on the computer screen, you
read:

man : I think that the hypothesis is indefensible. (A) She is unsure about the hypothesis.
woman : So do I. (B) The hippopotamus is behind the f ence.
narrator : Mat does the woman mean? (C) She thinks that the hypothesis can be
def ended.
(D) She agrees with the man.
The expression So do I is an expression that shows agreement with a positive statement, so the woman means that
she agrees with the man. The best answer is therefore answer (D). Other expressions are used to show agreement
with negative statements.

Example from the Paper and Computer TOEFL® Tests


On the recording, you hear: In your test book or on the computer screen, you
read:

woman : I don't think that our history teacher is (A) He disagrees with the woman.
very interesting. (B) He thinks the history teacher is
man : Neither do I. interesting.
narrator : What does the man mean? (C) He shares the woman's opinion.
(D) He doesn't think the woman's idea is
good.
The expression Neither do I is an expression that shows agreement with a negative statement, so the man shares the
woman's opinion. The best answer is therefore answer (C).

The following chart lists common expressions that show agreement. You should become familiar with these
expressions:
EXPRESSIONS OF AGREEMENT
Agreement with Positive Statements Agreement with Negative Statements
, i
So do I. Neither do I.
Me, too. I don't either.
I'll say!
Isn't it!
You can say that again!

EXERCISE 11: In this exercise, underline the expression of agreement in each short dialogue. Then read the
question and choose the best answer to that question. Remember that the best answer is one that shows agreement.

1. (woman) These paintings are really fascinating! (A) These paintings aren't very interesting.
(man) Aren’t they! (B) He isn't fascinated by these paintings.
(narrator) What does the man mean? (C) He isn't sure how he feels.
(D) He finds these paintings quite interesting.

2. (woman) I don't really care for the way me (A) He thinks the building was not renovated.
building was renovated. (B) He has the same opinion of the building as the
(man) I don't either. woman.
78
(narrator) What does the man mean? (C) He doesn't care about the renovation of the
building.
(D) He suggests being careful in the renovated
building.

3. (man) I think that both candidates for (A) She agrees with the man.
county supervisor is unqualified. (B) She thinks he should become county
(woman) Me, too. supervisor.
(narrator) What does the woman mean? (C) She thinks the candidates are qualified.
(D) She has no opinion about the candidates for
county supervisor.

TOEFL EXERCISE 11: In this exercise, listen carefully to each short dialogue and question on the recording, and
then choose the best answer to the question. You should pay attention to expressions of agreement.

NOW BEGIN THE RECORDING AT TOEFL EXERCISE 11.

1. (A) The trip would cost too much. (C) She would like to take two trips rather than one.
(B) She doesn't think that a trip would be (D) She would also like to take a trip.
a good idea.

2. (A) He would like to see the elections for town (C) He thinks the elections should take place next
council. month.
(B) He agrees that Matt should be elected. (D) He disagrees with the woman.

3. (A) She is not sure which course she should (C) She knows that she is not ready for intermediate
take. French.
(B) She's not sure if she should take a trip to (D) She wants to take neither beginning nor
France. Intermediate French.

4. (A) The man should repeat what he said. (C) She thinks that the food is the best she has
ever tasted.
(B) The man said something foolish. (D) She agrees that the food is pretty bad.

5. (A) This party hasn't been any fun at all. (C) He wants to know what she said.
(B) He wonders if the woman enjoyed herself. (D) He's enjoyed himself tremendously.

6. (A) She condones what happened. (C) She agrees with the man about what happened.
(B) She does not like what the man said. (D) She says that she did not do it.

7. (A) He thinks the parties aren't loud. (C) He agrees that the upstairs neighbors are noisy.
(B) He says that the neighbors don't have (D) The loud parties don't bother him.
many parties.

8. (A) She doesn't like this meal too much. (C) She's not sure if she likes it.
(B) This food tastes wonderful to her. (D) She can't stand this meal.

9. (A) She agrees that getting the car was not a (C) She thinks that the man is mistaken about the car.
good idea.
(B) She imagines that she would like to have a (D) She thinks the man has no imagination.
similar car.

10. (A) He would like the woman to repeat what (C) He also thinks that the course should be extended.
she said.
(B) He thinks that one semester is enough (D) He would like to take the course two semesters
time for the course. from now.

79
SKILL 12: LISTEN FOR EXPRESSIONS OF UNCERTAINTY AND SUGGESTION

Expressions of uncertainty and suggestion are common in the short dialogues, so you should become familiar with
them. The following example shows an expression of uncertainty.

Example from the Paper and Computer TOEFL® Tests


On the recording, you hear: In your test book or on the computer screen, you
read:

man : Do you know anything about the final (A) The exam is not going to be too
exam in physics? diff icult.
woman : It's going to be rather difficult, isn't it? (B) She's positive that it's going to be hard.
narrator : What does the woman mean? (C) She thinks that it might be hard.
(D) She has no idea about the exam.
The tag question isn't it changes a definite statement into a statement that shows uncertainty, so the best answer is
one that expresses uncertainty. The best answer to this question is answer (C) because the words thinks and might
express uncertainty. Other expressions that are common in the short dialogues are expressions of suggestion.

Example from the Paper and Computer TOEFL® Tests


On the recording, you hear: In your test book or on the computer screen, you
read:

man : I’ll never have time to type my paper (A) Finishing the paper today
tomorrow. (B) Not working on the paper now
woman : Why not do it now? (C) Never typing the paper
narrator : What does the woman suggest? (D) Taking time out f rom the paper now
In this example, the expression Why not is an expression of suggestion, so the woman suggests doing it now. In this
suggestion, the woman is referring to the paper that the man needs to type, so the best answer is answer (A).

The following chart lists common expressions that show uncertainty and suggestion:
EXPRESSIONS OF UNCERTAINTY AND SUGGESTION

Uncertainty Suggestion
...isn't it (tag)? Why not ...
As far as I know. Let's…
As far as I can tell.

EXERCISE 12: In this exercise, underline the expression of uncertainty or suggestion in each short dialogue. Then
read the question and choose the best answer to that question. Remember that the best answer is one that shows
uncertainty or suggestion.

1. (man) Do you know what time they're (A) She's not completely sure when they are leaving.
leaving for the city? (B) They are returning from the city at about 4:00.
(woman) They have to leave at four o'clock, (C) She knows when they are leaving.
don't they?
(narrator) What does the woman mean? (D) She doesn't have any idea when they are leaving.

2. (woman) I'm so thirsty from all this walking. (A) They should stop drinking.
(man) Let's stop and get a drink. (B) They should go for a walk.
(narrator) What does the man suggest? (C) They should walk thirty miles.
(D) They should take a break and have a drink.

80
3. (man) Is the exam still scheduled for 3:00 (A) The exam is far away.
on Thursday? (B) She knows that the exam schedule has been
(woman) As far as I know. changed.
(narrator) What does the woman mean? (C) She is sure that the exam is set for Thursday.
(D) She thinks she knows when the test is.

TOEFL EXERCISE 12: In this exercise, listen carefully to each short dialogue and question on the recording, and
then choose the best answer to the question. You should be particularly careful of expressions of uncertainty and
suggestion.

NOW BEGIN THE RECORDING AT TOEFL EXERCISE 12.

1. (A) He's sure about which chapters they are to (C) He has to tell her how far she should go.
read.
(B) He thinks he knows what the assignment (D) The professor told them to read the chapters
is. after the exam.

2. (A) The man should take the pie out. (C) The man shouldn't try cherry pie.
(B) The man should try something else. (D) The man should feel sorry.

3. (A) He knows the movie starts at 8:00. (C) He thinks the start of the movie has been changed.
(B) He is not quite sure when the movie (D) He will start the movie himself at 8:00.
begins.

4. (A) Not doing the dishes now (C) Leaving later so that they can do the dishes now
(B) Leaving the house with the dishes (D) Washing the dishes before they leave

5. (A) She's told Matt he'll go far. (C) She told Matt to roll farther.
(B) Matt has far from enough talent. (D) She believes Matt has the ability for the part.

6. (A) They should go to the hospital. (C) The woman should try not to break her leg.
(B) Mary should visit the man. (D) They should go on a trip with Mary.

7. (A) She knows where the children are. (C) She's going to the park to find the children.
(B) The children have finished playing ball. (D) She believes that the children are in the park.

8. (A) The man should try to borrow some from (C) The man should work on his math assignment
a neighbor. with Tom.
(B) The man should take a check to Tom. (D) The man should check behind the door.

9. (A) He thinks the bill is due in the middle of (C) He knows when they should pay the bill.
the month.
(B) The bill is approximately fifteen dollars. (D) The bill is going to be fifteen days late.

10.(A) They should postpone their decision until (C) They should not buy such a big house.
morning.
(B) They should go to sleep in the new house. (D) They should decide where to go to sleep.

SKILL 13: LISTEN FOR EMPHATIC EXPRESSIONS OF SURPRISE

Emphatic expressions of surprise are common in the short dialogues, so you should become familiar with them.
When surprise is expressed, it implies that the speaker did not expect something to be true.

81
Example from the Paper and Computer TOEFL® Tests
On the recording, you hear: In your test book or on the computer screen, you
read:

woman : Did you see Paul driving around in his (A) Paul would def initely get a Mustang.
Mustang? (B) Paul did not know how to drive.
man : Then he DID get a new car (C) Paul did not like Mustangs.
narrator : What had the man thought? (D) Paul would not get a new car.
In this dialogue the emphatic form he did get is used to show the man's surprise that Paul got a new car. It means
that the man expected that Paul would not get a new car, so the best answer is answer (D).

The following chart outlines various ways to express emphatic surprise:


EXPRESSIONS OF EMPHATIC SURPRISE

Verb Emphatic Form Example Meaning


be be, with emphasis Then he is here! I thought he was not here.
modal modal, with emphasis Then you can go! I thought you could not go
present tense do(es), with emphasis Then you do play tennis! I thought you did not play tennis
past tense did, with emphasis Then she did read it Then I thought she had not read it
perfect tense have, with emphasis he has gone there. I thought he had not gone there

EXERCISE 13: In this exercise, underline the expression of emphatic surprise in each short dialogue. Then read
the question and choose the best answer to that question. Remember that the best answer is one that shows surprise.

1. (man) I just got 600 on the TOEFL test! (A) The man had not passed.
(woman) Then you did pass. (B) The man would pass easily
(narrator) What had the woman assumed? (C) The man had already passed.
(D) The man got the score he was expected to get.

2. (woman) Would you like to go skiing this (A) The woman was a good skier
weekend? (B) The woman was going skiing this weekend.
(man) So you can ski! (C) The woman did not know how to ski.
(narrator) What had the man assumed? (D) The woman did not intend to go skiing.

3. (man) I just got this letter from my sister (A) The man's sister never wrote to him.
(woman) So the mail has come already. (B) The mail had not yet arrived.
(narrator) What had the woman assumed? (C) The mail always came early.
(D) The mail had already arrived.

TOEFL EXERCISE 13: In this exercise, listen carefully to each short dialogue and question on the recording, and
then choose the best answer to the question. You should be particularly careful of expressions of emphatic surprise.

NOW BEGIN THE RECORDING AT TOEFL EXERCISE 13.

1. (A) Greg always comes to parties. (C) Greg was unable to attend the party.
(B) Greg would come to the party later. (D) Greg would stay at the party for only a moment.

2. (A) The woman always rode her motorcycle to (C) The woman was an expert motorcycle rider.
school.
(B) The woman was not coming to school (D) The woman did not know how to ride a motorcycle.
today.
3. (A) The man was not a very good cook. (C) The man would never invite him over for dinner.
(B) The man never invited friends over for (D) The man was an excellent cook.
82
dinner.

4. (A) The woman had run more than three miles. (C) The woman ran for three hours in the morning.
(B) The woman always got lots of exercise. (D) The woman had not gotten much exercise.

5. (A) He had been somewhere else. (C) He had been working on his research project.
(B) He had been in the library. (D) He would start working on his project in five
hours.

TOEFL EXERCISE (Skills 11-13): In this exercise, listen carefully to each short dialogue and question on the
recording, and then choose the best answer to the question.

NOW BEGIN THE RECORDING AT TOEFL EXERCISE (SKILLS 11-13).

1. (A) She plans to talk a lot this month. (C) The bill is high because she has a lot to say.
(B) She has a lot to say about the phone bill. (D) She agrees with the man.

2. (A) Bill had never really been sick. (C) Bill was sick of calculus class.
(B) Bill was too sick to come to class. (D) Bill had forgotten about the calculus class that
morning.

3. (A) The man should go out tonight. (C) The man should work on the paper tonight.
(B) The man should stay home and relax. (D) The man should go out Monday instead.

4. (A) The cafeteria was open in the morning. (C) The breakfast in the cafeteria was not very tasty.
(B) The cafeteria did not serve breakfast. (D) The woman never ate breakfast in the cafeteria.

5. (A) He believes that it is acceptable to park (C) He knows that they won't get a ticket.
there.
(B) The parking lot is too far from their (D) He knows where the parking lot is.
destination.

83
TOEFL REVIEW EXERCISE (Skills 1-13): In this exercise, listen carefully to each short dialogue and question
on the recording, and then choose the best answer to the question.

NOW BEGIN THE RECORDING AT TOEFL REVIEW EXERCISE (SKILLS 1-13).

1. (A) Write a message to the man (C) Respond to the man's questions
(B) Make sonic phone calls (D) Get a new phone installed

2. (A) She's not sure if she's free. (C) She'll write a check for the calendar.
(B) She's marked it on her calendar. (D) Her calendar says she has to have a meeting
at 3:00.

3. (A) He barely rode the bicycle. (C) The bicycle didn't need to be paid for.
(B) He didn't have enough money. (D) He paid for the bicycle.

4. (A) She fixed the television. (C) The woman looked at Bob on television.
(B) Bob made the television work. (D) Bob works for the woman.

5. (A) He helped her say what she couldn't say. (C) He hasn't helped her very much.
(B) She was unable to say anything about him. (D) What he said was very helpful.

6. (A) The man should spend more time on (C) The man should send in his registration
registration. materials.
(B) The man should walk more quickly (D) The man should try to avoid registering
through registration. next semester.

7. (A) He couldn't find Paula's phone number, (C) When he went to call Paula, he couldn't find
so he didn't call her. the list.
(B) He couldn't give Paula the list over the (D) He couldn't recollect the number that was on
phone. the list.

8. (A) She couldn't take her luggage to the store. (C) She carried her luggage from the train station
to the store.
(B) She stored her luggage at the train station. (D) There were no lockers for her bags.

9. (A) The woman had taken a different major. (C) The woman was uninformed.
(B) The woman had chosen psychology as (D) The woman needed to see a psychiatrist.
a major.

10.(A) She would like the man to repeat what he (C) She shares the same opinion of the exam
said. as the man.
(B) She thinks the exam could have been a (D) She believes that the exam was easy.
little more difficult.

SKILL 14: LISTEN FOR WISHES

Conversations about wishes can appear in the short dialogues. The important idea to remember about wishes is that
a wish implies that the opposite of the wish is live.

Example from the Paper and Computer TOEFL® Tests


On the recording, you hear: In your test book or on the computer screen, you
read:

woman : It's too bad that you have to stay here and (A) Maybe he will go with the others on the
work during the school break. trip.
84
man : I really wish I could go with you and the (B) He is unable to go on the trip.
others to Palm Springs. (C) He's happy to be going on the trip.
narrator : What does the man mean? (D) He's going on the trip, but not with the
others.
In this dialogue the man wishes that he could go with the others on the trip, so the implied meaning is that he is
unable to go. The correct answer is therefore answer (B).

The following chart outlines the key points that you should know about wishes:

:. EY INFORMATION ABOUT WISHES


K

Point Example Meaning

 An affirmative wish implies a negative reality. I wish I had time to help. = no time to help =
 A negative wish implies an affirmative reality.
I wish I did not have time to help. time to help

 A past tense verb implies a present reality. I wish he were at home.* = is not at home
 A past perfect tense verb implies a past reality. I wish he had been at home. = was not at home

*Remember that were is used instead of was in wishes. I wish I were going.

EXERCISE 14: In this exercise, underline the wish in each short dialogue. Then read the question and choose the
best answer to that question. Remember that the best answer is one that implies the opposite of what is said.

1. (man) Do you think we’ll be able to have (A) The sky is not very cloudy.
the picnic today? (B) The sky yesterday was cloudier than it is today.
(woman) I wish the sky weren't so cloudy. (C) The sky is too cloudy.
(narrator) What does the woman mean? (D) The sky is rather clear.

2. (woman) Did you enjoy the Thanksgiving (A) He didn't eat very much.
dinner? (B) He plans on eating a lot.
(man) I wish I hadn't eaten so much. (C) He thinks he is eating a lot.
(narrator) What does the man mean? (D) He ate too much.

3. (man) Are you coming to the party (A) She is coming to the party.
tonight? (B) She might come to the party.
(woman) I wish I could. (C) She will try to come to the party.
(narrator) What does the woman mean? (D) She is not coming to the party.

TOEFL EXERCISE 14: In this exercise, listen carefully to each short dialogue and question on the recording, and
then choose the best answer to the question. You should remember that a wish implies an opposite meaning.

NOW BEGIN THE RECORDING AT TOEFL EXERCISE 14.

1. (A) The line is short. (C) The line in front of them is too long.
(B) There are not very many people in front (D) Not many people want to get tickets to the
of them. concert.

2. (A) The woman told him about the ticket. (C) He was happy to find out about the ticket.
(B) He wanted the woman to get a ticket. (D) The woman did not tell him about the ticket.

85
3. (A) She is not working too many hours next (C) She is working too many hours next week.
week.
(B) She doesn't have enough hours next week. (D) She likes working so much.

4. (A) The department did not change the (C) She changed her apartment just before
requirements. graduation.
(B) She likes the new requirements. (D) She does not like the changes that the department
made.

5. (A) He is going to the theater. (C) He isn't afraid to go.


(B) He doesn't have enough money. (D) He doesn't want to spend the money.

SKILL 15: LISTEN FOR UNTRUE CONDITIONS

Conversations containing conditions can appear in the short dialogues. The important idea to remember about
conditions is that a condition implies that the opposite of the condition is true.

Example from the Paper and Computer TOEFL® Tests


On the recording, you hear: In your test book or on the computer screen, you
read:

man : Do you think that you'll be able to go to (A) Maybe she'll go.
the party? (B) She has time, so she'll go.
woman : If I had time, I would go. (C) She is going even if she doesn't have
narrator : What does the woman say about the time.
party? (D) It's impossible to go.
In this question, the condition If I had time implies that the opposite is true: The woman does not have time for the
party, so it's impossible to go. Therefore, the best answer to this question is answer (D).

The following box outlines the key points that you should know about untrue conditions:
INFORMATION ABOUT UNTRUE CONDITIONS

POINT EXAMPLE MEANING


 An affirmative condition implies a If she were at home, she could do it.* = not at home
negative reality.
 A negative condition implies an If she weren’t at home, she could do it. = at home
affirmative reality.
 A past tense implies a present reality. If I had money, I would buy it. = do not have money
 A past perfect verb implies a past
reality. If I had had money, I would have bought it. = did not have money
 Had can be used without if. Had I had money, I would have bought it.** = did not have money
*Remember that were is used instead of was in untrue conditions: "If I were there, I would help."
**This has the same meaning as "If I had had money...." Note that the subject and "had" are inverted.

EXERCISE 15: In this exercise, underline the condition in each short dialogue. Then read the question and choose
the best answer to that question. Remember that the best answer is one that implies the opposite of what is said.

1. (man) Are you going to have something (A) She is not going to eat.
to eat? (B) The food looks fresh.
(woman) If the food looked fresh, I would (C) She doesn't like fresh food.

86
eat some.
(narrator) What does the woman mean? (D) She already ate something.

2. (woman) The flight must have taken longer (A) It arrived early.
than usual. (B) It was unusually short.
(man) Had the flight left on time, we (C) It left on time.
would not have arrived so late. (D) It departed late.
(narrator) What does the man say about the
flight?

3. (man) Are you sure you want to go out? (A) She really is feeling fine.
You do not seem to be feeling very (B) There is plenty of aspirin in the medicine cabinet.
well. (C) It is necessary to get some aspirin.
(woman) If there were some aspirin in the (D) She does not need to go out.
medicine cabinet, I would not need
to go to the drugstore.
(narrator) What does the woman mean?

TOEFL EXERCISE 15: In this exercise, listen carefully to each short dialogue and question on the recording, and
then choose the best answer to the question. You should be particularly careful of untrue conditions.

NOW BEGIN THE RECORDING AT TOEFL EXERCISE 15.

1. (A) The woman did not need to call him. (C) He's not glad that the woman called.
(B) The woman called to let him know about (D) He already knew about the meeting when
the meeting. the woman called.

2. (A) The man often drives too quickly. (C) The man drove rather slowly.
(B) The police do not stop the man too much. (D) The police should not stop the man so often.

3. (A) She's so happy they don't have to work on (C) She wonders if the man would be nice enough to
Friday. come in to work in her place on Friday.
(B) It would be nice if they could finish their (D) It's too bad they must work on Friday.
work on Friday.

4. (A) She did not put enough postage on the (C) The letter did not need more postage.
letter.
(B) The letter arrived last week. (D) She did not put any postage on the letter.

5. (A) He has a dog. (C) He wishes he had a dog.


(B) He doesn't pay attention to dogs. (D) Dogs do not need much attention.

TOEFL EXERCISE (Skills 14-15): In this exercise, listen carefully to each short dialogue and question on the
recording, and then choose the best answer to the question.

NOW BEGIN THE RECORDING AT TOEFL EXERCISE (SKILLS 14-15).

1. (A) She enjoys violent movies. (C) She thinks the film was too violent.
(B) She would have preferred a more violent (D) She enjoyed the movie.
movie.
2. (A) He left the windows open. (C) He forgot to close the windows.
(B) The rain did not get in. (D) The rain got into the house.

3. (A) Her family is unable to come to graduation. (C) Her parents are coming to the ceremonies.
(B) It is possible that her family will come. (D) She is not graduating this year.

87
4. (A) He is going to miss the conference. (C) He will attend the conference.
(B) He will take his vacation next week. (D) He won't miss his vacation.

5. (A) He enjoys chemistry lab. (C) He isn't taking chemistry class.


(B) He doesn't have chemistry lab this (D) He has to go to the lab.
afternoon.

6. (A) They filled up the gas tank at the last (C) Even though they didn't stop at the service
service station. station, they didn't run out of gas.
(B) Although they filled up the tank, they still (D) They ran out of gas because they didn't stop at
ran out of gas. the gas station.

7. (A) His schedule is not really heavy. (C) He enrolled in more courses than he really wants.
(B) He needs to add a few more courses. (D) He will register for a lot of courses next
semester.

8. (A) She never took the bus to work. (C) She doesn't know how to get to work.
(B) She regularly takes the bus. (D) She gets lost on the bus.

9. (A) She bought some eggs at the store. (C) He can borrow some eggs.
(B) She doesn't have any eggs to lend him. (D) She didn't go to the store.

10.(A) Teresa is feeling a lot better. (C) Teresa didn't follow the doctor's orders.
(B) The doctor didn't prescribe the medicine. (D) Teresa did exactly what the doctor said.

TOEFL REVIEW EXERCISE (Skills 1-15): In this exercise, listen carefully to each short dialogue and question
on the recording, and then choose the best answer to the question.

NOW BEGIN THE RECORDING AT TOEFL REVIEW EXERCISE (SKILLS 1-15).

1. (A) Drinking the hot tea (C) Letting the tea cool off a bit
(B) Making more tea in a few minutes (D) Having the tea immediately

2. (A) In a bus station (C) In a restaurant


(B) In a store (D) In a theater

3. (A) He's unhappy to end the semester. (C) He couldn't be happier to begin the semester.
(B) He's glad to be finishing school. (D) The end of the semester is making him feel sad.

4. (A) The storm destroyed the house. (C) The stormy weather caused the trees to fall.
(B) The house blocked the trees. (D) During the storm, someone knocked on the door
of the house.

5. (A) The team hasn't won often. (C) It's out of the ordinary for the team to lose.
(B) He usually doesn't pay attention to the (D) He usually hears about the football games.
football team.

6. (A) He went to the office every morning. (C) He had to arrive at work earlier than 8 o'clock.
(B) He was not working. (D) He had a job.

7. (A) He did not enjoy his vacation as much as (C) The vacation was really enjoyable.
possible.
(B) He got lost on his vacation. (D) He did not really lose his passport.

88
8. (A) It will take eight hours to get to Riverdale (C) He doesn't know where Riverdale is.
on the bus.
(B) He believes he knows the correct bus. (D) He assures the woman that he knows the way to
Riverdale.

9. (A) The laboratory assistant completed one (C) The laboratory assistant didn't want to do more
experiment. experiments.
(B) The laboratory assistant couldn't finish (D) None of the experiments could be completed.
one experiment.

10.(A) She would like the man to repeat what he (C) The semester will never end.
said.
(B) The semester is really over! (D) She has the same wish as the man.

89
REFERENCES

Alavi, S. M., & Bordbar, S. (2012a). Are they right participants for the right strategies? A case study in the role
of levels of language ability in strategy use in reading section of TOEFL iBT®. Theory & Practice in
Language Studies, 2, 877–886. https://doi.org/10.4304/tpls.2.5.877-886

Alavi, S. M., & Bordbar, S. (2012b). A closer look at reading strategy use in reading section of TOEFL iBT.
Theory & Practice in Language Studies, 2, 450–460. https://doi.org/10.4304/tpls.2.3.450-460

Alderson, J. C. (2009). Test of English as a Foreign Language™: Internet-based Test (TOEFL® iBT). Language
Testing, 26, 621–631. https://doi.org/10.1177/0265532209346371

Baron, P. A., & Papageorgiou, S. (2014). Mapping the TOEFL® Primary™ Test onto the Common European
Framework of Reference (Research Memorandum No. RM14-05). Princeton, NJ: Educational Testing
Service.

Baron, P. A., & Papageorgiou, S. (2016). Setting language proficiency score requirements for English-as-a-
second-language placement decisions in secondary education (Research Report No. RR-16-17).
Princeton, NJ: Educational Testing Service. https://doi.org/10.1002/ets2.12102

Baron, P. A., & Tannenbaum, R. J. (2011). Mapping the TOEFL Junior® test onto the Common European
Framework of Reference (Research Memorandum No. RM11-07). Princeton, NJ: Educational Testing
Service.

ETS. (2010a). Linking TOEFL iBT™ Scores to IELTS Scores - A Research Report, 1-17. Retrieved from
http://www.ets.org/s/toefl/pdf/linking_toefl_ibt_scores_to_ielts_scores.pdf

ETS. (2010b). TOEFL iBT™ Test Framework and Test Development. TOEFL iBT™ Research Insight, 1(1), 1-8.
Retrieved from http://www.ets.org/s/toefl/pdf/toefl_ibt_research_insight.pdf

ETS. (2010c). TOEFL Research. TOEFL iBT™ Research Insight, 1(2), 1-8. Retrieved from
http://www.ets.org/s/toefl/pdf/toefl_ibt_insight_s1v2.pdf

ETS. (2011a). Reliability and Comparability of TOEFL iBT™ Scores. TOEFL iBT™ Research Insight, 1(3), 1-
8. Retrieved from http://www.ets.org/s/toefl/pdf/toefl_ibt_research_s1v3.pdf

ETS. (2011b). Validity Evidence Supporting the Interpretation and Use of TOEFL iBT™ Scores. TOEFL iBT™
Research Insight, 1(4), 1-12. Retrieved from http://www.ets.org/s/toefl/pdf/toefl_ibt_insight_s1v4.pdf

Gallagher, Nancy. (2000). Delta’s Key to the TOEFL Test. Delta Publishing.

Phillips, Deborah. (2001). Complete Course for the TOEFL Test. New York: Longman.

Pyle, Michael A., M.A. Page, Mary Ellen Munoz. (1995). TOEFL Preparation Guide. Nebraska: Cliff Notes
Incorporates.

90

You might also like